Você está na página 1de 50

Manual de soluções para

Funções de One I Variável Complexa, segunda edição 1

© Copyright by Andreas Kleefeld de 2009


Todos os direitos reservados 2

1 por John B. Conway


2 Última Atualização em abril de 15 2016
PREFÁCIO

A maioria dos exercícios eu resolvi foram atribuídos trabalhos de casa nos cursos de pós-graduação de matemática 713 e Matemática 714 “Análise Complexa
I e II”, da Universidade de Wisconsin - Milwaukee ministrado pelo Professor Dashan Fan no Outono de 2008 e Primavera de 2009.

O manual de soluções destina-se a todos os estudantes que um curso de Análise Complexa nível de pós-graduação. mossas estu- pode verificar suas
respostas para problemas de casa atribuídos a partir do excelente livro “funções de uma variável I Complex”, segunda edição por John B. Conway.
Além disso os alunos podem se preparar para testes, testes, exames e exames fi nal por resolver exercícios adicionais e verificar seus resultados.
Talvez os alunos, mesmo estudar para os exames preliminares para seus estudos de doutoramento.

No entanto, eu tenho que avisá-lo para não copiar reta deste livro e vire em sua casa, porque isso violaria a finalidade de trabalhos de
casa. Claro, isso é com você. Encorajo vivamente a SendMe soluções que ainda faltam para a.kleefeld@fz-juelich.de ( eu UMA TEXpreferred
mas não obrigatório), a fim de completar este manual de soluções. Pense sobre a contribuição que você vai dar a outros estudantes.

Se você achar erros de digitação ou erros matemáticos pop um email para a.kleefeld@fz-juelich.de. A versão mais recente deste manual soluções
podem ser encontradas em
http://www.math.tu-cottbus.de/INSTITUT/kleefeld/Files/Solution.html.
O objectivo deste projecto é dar soluções a todos os 452 exercícios.

CONTRIBUIÇÃO

Agradeço (sem ordem especial)

Christopher T. Alvin Martin J. Michael


David Perkins Robert Browning para
contribuições para este livro.

2
Conteúdo

1 O Sistema de número complexo 1


1.1 Os números reais . . . . . . . . . . . . . . . . . . . . . . . . . . . . . . . . . . . . . . . 1
1.2 o campo de números complexos . . . . . . . . . . . . . . . . . . . . . . . . . . . . . . . . 1
1.3 O plano complexo . . . . . . . . . . . . . . . . . . . . . . . . . . . . . . . . . . . . . . 5
1,4 representação polar e raízes de números complexos . . . . . . . . . . . . . . . . . . . . . 5
Linhas 1,5 e semiplanos no plano complexo . . . . . . . . . . . . . . . . . . . . . . . . . . 7
1.6 O plano estendido e a sua representação esférica . . . . . . . . . . . . . . . . . . . . . 8

2 espaços métricos e a topologia de C 10


2.1 As definições e exemplos de espaços métricos . . . . . . . . . . . . . . . . . . . . . . . . . . 10
2.2 Connectedness . . . . . . . . . . . . . . . . . . . . . . . . . . . . . . . . . . . . . . . . 13
2.3 integralidade e Sequências . . . . . . . . . . . . . . . . . . . . . . . . . . . . . . . . . 14
2.4 Compacidade . . . . . . . . . . . . . . . . . . . . . . . . . . . . . . . . . . . . . . . . . 16
2,5 Continuidade . . . . . . . . . . . . . . . . . . . . . . . . . . . . . . . . . . . . . . . . . . . 18
2.6 Convergência uniforme . . . . . . . . . . . . . . . . . . . . . . . . . . . . . . . . . . . . . 21

3 Propriedades elementar e Exemplos das funções analíticas 22


série 3.1 Poder . . . . . . . . . . . . . . . . . . . . . . . . . . . . . . . . . . . . . . . . . . 22
3.2 As funções analíticas . . . . . . . . . . . . . . . . . . . . . . . . . . . . . . . . . . . . . . . 25
3.3 As funções analíticas como mapeamentos. transformações de Möbius . . . . . . . . . . . . . . . . . . 32

4 Integração Complex 43
4.1 integrais de Riemann-Stieltjes . . . . . . . . . . . . . . . . . . . . . . . . . . . . . . . . . . 43
representação em série de 4,2 de energia das funções analíticas . . . . . . . . . . . . . . . . . . . . . . . 49
4,3 zeros de uma função analítica . . . . . . . . . . . . . . . . . . . . . . . . . . . . . . . . . 59
4.4 O índice de uma curva fechada . . . . . . . . . . . . . . . . . . . . . . . . . . . . . . . . . . 61
4,5 Teorema de Cauchy e fórmula integral . . . . . . . . . . . . . . . . . . . . . . . . . . . 62
4.6 A versão homotópicas do Teorema de Cauchy e conectividade simples . . . . . . . . . . . 64
4.7 zeros contagem; Open Mapeamento Teorema . . . . . . . . . . . . . . . . . . . . . . . . 67
4.8 Teorema de Goursat . . . . . . . . . . . . . . . . . . . . . . . . . . . . . . . . . . . . . . 68

5 Singularidades 69
5.1 Classi fi cação de singularidades . . . . . . . . . . . . . . . . . . . . . . . . . . . . . . . . . 69
5.2 Resíduos . . . . . . . . . . . . . . . . . . . . . . . . . . . . . . . . . . . . . . . . . . . . 76
5.3 O principal argumento . . . . . . . . . . . . . . . . . . . . . . . . . . . . . . . . . . . 83

3
6 O MaximumModulus Teorema 85
6.1 Princípio A Máxima . . . . . . . . . . . . . . . . . . . . . . . . . . . . . . . . . . . 85
6.2 Lema de Schwarz . . . . . . . . . . . . . . . . . . . . . . . . . . . . . . . . . . . . . . . 87
6.3 funções convexas e três círculos Teorema de Hadamard . . . . . . . . . . . . . . . . . 89
6.4 O Phragmén-Lindelöf Teorema . . . . . . . . . . . . . . . . . . . . . . . . . . . . . . 91

7 compacidade e Convergência no espaço de Funções Analíticas 94


7.1 O espaço de funções contínuas CG, Ω) . . . . . . . . . . . . . . . . . . . . . . . . . 94
7,2 espaços de funções analíticas . . . . . . . . . . . . . . . . . . . . . . . . . . . . . . . . . 98
7.3 Espaços de funções meromorfas . . . . . . . . . . . . . . . . . . . . . . . . . . . . . . 101
7.4 A Riemann Mapeamento Teorema . . . . . . . . . . . . . . . . . . . . . . . . . . . . . . . 104
7.5 A Weierstrass Factorization Teorema . . . . . . . . . . . . . . . . . . . . . . . . . . . 107
7,6 Factorization da função seno . . . . . . . . . . . . . . . . . . . . . . . . . . . . . . . 111
7.7 A função gama . . . . . . . . . . . . . . . . . . . . . . . . . . . . . . . . . . . . . 113
7.8 A função Riemann zeta . . . . . . . . . . . . . . . . . . . . . . . . . . . . . . . . . . 121

8 Teorema de Runge 124


8.1 Teorema de Runge . . . . . . . . . . . . . . . . . . . . . . . . . . . . . . . . . . . . . . . 124
8.2 conexão simples . . . . . . . . . . . . . . . . . . . . . . . . . . . . . . . . . . . . . 125
8,3 Mittag-Le ffl Teorema de er . . . . . . . . . . . . . . . . . . . . . . . . . . . . . . . . . . . 126

9 continuação analítica e Riemann Superfícies 131


9.1 Schwarz Re Princípio fl exão . . . . . . . . . . . . . . . . . . . . . . . . . . . . . . . . . 131
9.2 Continuação Analytic longo de um caminho . . . . . . . . . . . . . . . . . . . . . . . . . . . . . 132
9,3 Monodromia Teorema . . . . . . . . . . . . . . . . . . . . . . . . . . . . . . . . . . . . . 133
9,4 espaços topológicos e Sistemas Bairro . . . . . . . . . . . . . . . . . . . . . . . 133
9.5 The Sheaf of Germs of Analytic Functions on an Open Set . . . . . . . . . . . . . . . . . 134
9.6 Analytic Manifolds . . . . . . . . . . . . . . . . . . . . . . . . . . . . . . . . . . . . . . 135
9.7 Covering spaces . . . . . . . . . . . . . . . . . . . . . . . . . . . . . . . . . . . . . . . . 136

10 Harmonic Functions 138


10.1 Basic properties of harmonic functions . . . . . . . . . . . . . . . . . . . . . . . . . . . . 138
10.2 Harmonic functions on a disk . . . . . . . . . . . . . . . . . . . . . . . . . . . . . . . . . 142
10.3 Subharmonic and superharmonic functions . . . . . . . . . . . . . . . . . . . . . . . . . . 145
10.4 The Dirichlet Problem . . . . . . . . . . . . . . . . . . . . . . . . . . . . . . . . . . . . 149
10.5 Green’s Function . . . . . . . . . . . . . . . . . . . . . . . . . . . . . . . . . . . . . . . 149

11 Entire Functions 152


11.1 Jensen’s Formula . . . . . . . . . . . . . . . . . . . . . . . . . . . . . . . . . . . . . . . 152
11.2 The genus and order of an entire function . . . . . . . . . . . . . . . . . . . . . . . . . . 154
11.3 Hadamard Factorization Theorem . . . . . . . . . . . . . . . . . . . . . . . . . . . . . . 159

12 The Range of an Analytic Function 162


12.1 Bloch’s Theorem . . . . . . . . . . . . . . . . . . . . . . . . . . . . . . . . . . . . . . . 162
12.2 The Little Picard Theorem . . . . . . . . . . . . . . . . . . . . . . . . . . . . . . . . . . 163
12.3 Schottky’s Theorem . . . . . . . . . . . . . . . . . . . . . . . . . . . . . . . . . . . . . . 163
12.4 The Great Picard Theorem . . . . . . . . . . . . . . . . . . . . . . . . . . . . . . . . . . 163

4
Chapter 1

The Complex Number System

1.1 The real numbers


No exercises are assigned in this section.

1.2 The field of complex numbers


Exercise 1. Find the real and imaginary parts of the following:

• •3
1 •• − 1 + i √ 3 ••
•• •• ;
• •
z ; zz
−a+ a(a ∈ R); z 3; 3 + 5 i7 i + 1 ; 2

• •6 ( 1 + i )n
•• − 1 − i √ 3 ••
•• •• ; i n;
• • √2 for 2 ≤ n ≤ 8.
2

Solution. Let z = x + iy. Then


a)
(1 )
x x 2 + y 2 = Re(z)
Re =
z | z| 2
(1 )
Im =−y
z x 2 + y 2 = − Im(z) | z| 2

b)

)
x2+ y2− a2 | z| 2 − a 2
Re ( z − a =
z+a x 2 + y 2 + 2 ax + a 2 = | z| 2 + 2 aRe(z) + a 2
)
2 ya x 2 + y 2 + 2 xa + 2 Im(z)a
Im ( z − a = a2=
z+a | z| 2 + 2 aRe(z) + a 2

c)

Re (z 3) = x 3 − 3 xy 2 = Re(z) 3 − 3 Re(z)Im(z) 2

Im (z 3) = 3 x 2 y − y 3 = 3 Re(z) 2 Im(z) − Im(z) 3

1
d)
(3+5i )
Re = 19
7i+1 25
(3+5i )
Im =−8
7i+1 25

e)
•• • 3 •••••••• = 1
•• • − 1 + i √ 3 ••
••
Re ••• ••• ••

• 2
•• • 3 •••••••• = 0
•• • − 1 + i √ 3 ••
••
Im •••• ••• ••

2

f)
•• • 6 •••••••• = 1
•• • − 1 − i √ 3 ••
••
Re ••• ••• ••

• 2
•• • 6 •••••••• = 0
•• • − 1 − i √ 3 ••
••
Im •••• ••• ••

2

g)

•• 0, n is odd
••
••
Re (i n) = •• 1, n ∈ { 4 k : k ∈ Z}
••
••
− 1, n ∈ { 2 + 4 k : k ∈ Z}

•• 0, n is even
••
••
Im (i n) = •• 1, n ∈ { 1 + 4 k : k ∈ Z}
••
••
− 1, n ∈ { 3 + 4 k : k ∈ Z}

2
h)

•• 0, n=2
••
••
••
•• − √ 22 , n = 3
••
••
(( 1 + i ) n) •• − 1, n=4
••

Re √2 = •• − √ 22 , n = 5
••
••
•• 0, n=6
•• √2
••
•• n=7
•• 2,
••
• 1, n=8

•• 1, n=2
••
•• √2
••
•• n=3
•• 2,
••
(( 1 + i ) n) •• 0, n=4
••

Im √2 = •• − √ 22 , n = 5
••
••
•• − 1, n=6
••
••
•• − √ 22 , n = 7
••
••
• 0, n=8

Exercise 2. Find the absolute value and conjugate of each of the following:

i i + 3 ; (1 + i) 6; i 17.
− 2 + i; − 3; (2 + i)( 4 + 3 i); 3 − i √ 2 + 3 i ;

Solution. It is easy to calculate:


a)
z = − 2 + i, | z| = √ 5, zz̄ = − 2 − i

b)
z = − 3, | z| = 3, zz̄ = − 3

c)
z = ( 2 + i)( 4 + 3 i) = 5 + 10 i, | z| = 5 √ 5, zz̄ = 5 − 10 i

d
√ 110,
z = 3 −√i 2 + 3 i , | z| = 1 zz̄ = 3 +√ i2 − 3 i
11

e)
√ 10,
z=i | z| = 1 zz̄ = 1
i + 3 = 1 10 + 3 10 i, 10 10 − 3 10 i

f)
z = ( 1 + i) 6 = − 8 i, | z| = 8, zz̄ = 8 i

g)
i 17 = i, | z| = 1, zz̄ = − i

Exercise 3. Show that z is a real number if and only if z = ¯ z.

3
Solution. Let z = x + iy.
⇒ : If z is a real number, then z = x (y = 0). This implies ¯ z = x and therefore z = ¯ z.
⇔ : If z = ¯ z, then we must have x + iy = x − iy for all x, y ∈ R. This implies y = − y which is true if y = 0 and therefore z = x. This means
that z is a real number.

Exercise 4. If z and w are complex numbers, prove the following equations:

| z + w| 2 = | z| 2 + 2 Re(z ¯ w) + |w| 2.
| z − w| 2 = | z| 2 − 2 Re(z ¯ w) + |w| 2.
| z + w| 2 + | z − w| 2 = 2 (| z| 2 + | w| 2) .

Solution. We can easily verify that ¯¯ z = z. Thus

| z + w| 2 = ( z + w)(z + w) = (z + w)( ¯ z + ¯w) = z ¯ z + z ¯ w + w ¯ z + w ¯ w

= | z| 2 + | w| 2 + z ¯ w + ¯ zw = |z| 2 + | w| 2 + z ¯ w + ¯z w
¯¯

w+z¯ w
= | z| 2 + | w| 2 + z ¯ w+z¯ w = |z| 2 + | w| 2 + 2 z ¯
2
= | z| 2 + | w| 2 + 2 Re(z ¯ w) = |z| 2 + 2 Re(z ¯ w) + |w| 2.

| z − w| 2 = ( z − w)(z − w) = (z − w)( ¯ z − ¯ w) = z ¯ z − z ¯ w − w ¯ z + w ¯ w
= | z| 2 + | w| 2 − z ¯ w − ¯ zw = |z| 2 + | w| 2 − z ¯ w − ¯ zw
¯¯

w+z¯ w
= | z| 2 + | w| 2 − z ¯ w − z ¯ w = |z| 2 + | w| 2 − 2 z ¯
2
= | z| 2 + | w| 2 − 2 Re(z ¯ w) = |z| 2 − 2 Re(z ¯ w) + |w| 2.

| z + w| 2 + | z − w| 2 = | z| 2 + Re(z ¯ w) + |w| 2 + | z| 2 − Re(z ¯ w) + |w| 2

= | z| 2 + | w| 2 + | z| 2 + | w| 2 = 2| z| 2 + 2| w| 2 = 2 (| z| 2 + | w| 2) .

Exercise 5. Use induction to prove that for z = z 1 + . . . + z n; w = w 1 w 2 . . . w n:

| w| = |w 1| . . . | w n|; ¯ z = ¯z 1 + . . . + ¯ z n; ¯w = ¯ w 1 . . . ¯ w n.

Solution. Not available.

Exercise 6. Let R(z) be a rational function of z. Show that R(z) = R( ¯ z) if all the coe ffi cients in R(z) are real.

Solution. Let R(z) be a rational function of z, that is

R(z) = a n z n + a n − 1 z n − 1 + . . . a 0
bm zm+ bm−1 zm−1+... b0

where n, m are nonnegative integers. Let all coe ffi cients of R(z) be real, that is

a 0, a 1, . . . , a n, b 0, b 1, . . . , b m ∈ R.

Then

R(z) = a n z n + a n − 1 z n − 1 + . . . a 0 = an zn+ an−1 zn−1+... a0


bm zm+ bm−1 zm−1+... b0 bm zm+ bm−1 zm−1+... b0

z n + a n − 1 ¯z n − 1 + . . . a 0
= an zn+ an−1 zn−1+... a0 = an ¯ = R( ¯ z).
bm zm+ bm−1 zm−1+... b0 b m ¯z
z̄ m + b m − 1 ¯ z m − 1 + . . . b 0

4
1.3 The complex plane
Exercise 1. Prove (3.4) and give necessary and su ffi cient conditions for equality.

Solution. Let z and w be complex numbers. Then

|| z| − | w|| = ||z − w + w| − | w||

≤ || z − w| + |w| − | w||

= || z − w||

= | z − w|

Notice that |z| and |w| is the distance from z and w, respectively, to the origin while |z − w| is the distance between z and w.
Considering the construction of the implied triangle, in order to guarantee equality, it is necessary and su ffi cient that

|| z| − | w|| = |z − w|

⇐ ⇒ (| z| − | w|) 2 = | z − w| 2

⇐ ⇒ (| z| − | w|) 2 = | z| 2 − 2 Re(z ¯ w) + |w| 2

⇐ ⇒ | z| 2 − 2| z||w| + |w| 2 = | z| 2 − 2 Re(z ¯ w) + |w| 2

⇐ ⇒ | z||w| = Re(z ¯ w)
⇐ ⇒ | z w|
w̄|
¯ = Re(z ¯ w)

Equivalently, this is z ¯ w ≥ 0. Multiplying this by ww , we get z ¯ w·w


w| 2 · z w≥ 0 if w , 0. If
(1 ) w=|

t =z w= | w| 2
· | w| 2 · z
w. Then t ≥ 0 and z = tw.
Exercise 2. Show that equality occurs in (3.3) if and only if z k/ z l ≥ 0 for any integers k and l, 1 ≤ k, l ≤ n, for which z l , 0.

Solution. Not available.

Exercise 3. Let a ∈ R and c > 0 be fixed. Describe the set of points z satisfying

| z − a| − | z + a| = 2 c

for every possible choice of a and c. Now let a be any complex number and, using a rotation of the plane, describe the locus of points
satisfying the above equation.

Solution. Not available.

1.4 Polar representation and roots of complex numbers


Exercise 1. Find the sixth roots of unity.

Solution. Start with z 6 = 1 and z = r cis( θ), therefore r 6 cis(6 θ) = 1. Hence r = 1 and θ = 2 k π 6 with k ∈
{− 3, − 2, − 1, 0, 1, 2}. The following table gives a list of principle values of arguments and the corresponding value of the root of the
equation z 6 = 1.
θ0= 0 z0= 1
θ1=π3 z 1 = cis( π 3 )
θ2=2π 3
z 2 = cis( 2 π 3)
θ3= π z 3 = cis( π) = − 1
θ 4 = − 2 π3 z 4 = cis( − 2 π 3 )
θ5=−π 3
z 5 = cis( − π 3)

5
Exercise 2. Calculate the following:

a) the square roots of i

b) the cube roots of i

c) the square roots of √ 3 + 3 i

Solution. c) The square roots of √ 3 + 3 i. Let z = √ 3 + 3 i.


√( √ 3) 2 + 3 2 = √ 12 and α = tan − 1 ( 3 )
Then r = |z| = √3 = π 3 . So, the 2 distinct roots of z
√ r ( cos α+ 2 k π )
are given by 2 n+ i sin α+ 2 k π n
where k = 0, 1. Specifically,

( )
√ z = 4 √ 12
cos π 3 + 2 k π .
2 + i sin π 3 + 2 k π 2

Therefore, the square roots of z, z k, are given by


√ 12 (cos π 6 + i sin π 6 ) √ 12 ( √ 3
z 0 =4 =4
2 + 12 i)
√ 12 (cos 7 π ) √ 12 ( − √ 3
z 1 =4 6+ i sin 7 π 6
=4
2 − 12 i).

√ 108 √ 12 2) √ 108 √ 12 2)

So, in rectangular form, the second roots of z are given by ( 4 2 ,4


and ( − 4 2, −4 .

Exercise 3. A primitive nth root of unity is a complex number a such that 1, a, a 2, ..., a n − 1 are distinct nth roots of unity. Show that if a and
b are primitive nth and mth roots of unity, respectively, then ab is a kth root of unity for some integer k. What is the smallest value of
k? What can be said if a and b are nonprimitive roots of unity?

Solution. Not available.

Exercise 4. Use the binomial equation

(nk )
∑n
( a + b) n = a n − k b k,
k= 0

where (nk )
n! k!(n − k)!
= ,

and compare the real and imaginary parts of each side of de Moivre’s formula to obtain the formulas:
) (n4 )

cos n θ = cos n θ − ( n 2 cos n − 2 θ sin 2 θ + cos n − 4 θ sin 4 θ − . . .

(n1 ) )
sin n θ = cos n − 1 θ sin θ − ( n 3 cos n − 3 θ sin 3 θ + . . .

Solution. Not available.

Exercise 5. Let z = cis 2 π n for an integer n ≥ 2. Show that 1 + z + . . . + z n − 1 = 0.

6
Solution. The summation of the finite geometric sequence 1, z, z 2, . . . , z n − 1 can be calculated as ∑ nj= 1 z j − 1 =
( cis ( 2 π ) ) n =
zn − 1
z−1. We want
) to show that z n is an n th root of unity. So, using de Moivre’s formula, z n = n

cis (n · 2 π
n
= cis( 2 π) = 1. It follows that 1 + z + z 2 + ... + z n − 1 = z n − 1 z − 1 = 1 − 1z − 1 = 0 as required.

Exercise 6. Show that ϕ( t) = cis t is a group homomorphism of the additive group R onto the multiplicative group T = {z : |z| = 1}.

Solution. We have

ϕ( s + t) = cos( s + t) + i sin( s + t)

= [ cos( s) cos( t) − sin( s) sin( t)] + i [ sin( s) cos( t) + cos( s) sin( t)]

= cos( s) cos( t) + i cos( s) sin( t) + i sin( s) cos( t) − sin( s) sin( t)

= ( cos( s) + i sin( s)) ( cos( t) + i sin( t))

= ϕ( s) · ϕ( t) , ∀ s, t ∈ R

and | ϕ( t)| = 1 for any t ∈ R.

Exercise 7. If z ∈ C and Re(z n) ≤ 0 for every positive integer n, show that z is a non-negative real number.

Solution. Let n be an arbitrary but fixed positive integer and let z ∈ C and Re(z n) ≥ 0. Since z n =
r n( cos( n θ) + i sin( n θ)), we have
Re(z n) = r n cos( n θ) ≥ 0.

If z = 0, then we are done, since r = 0 and Re(z n) = 0. Therefore, assume z , 0, then r > 0. Thus

Re(z n) = r n cos( n θ) ≥ 0

implies cos( n θ) ≥ 0 for all n. This implies θ = 0 as we will show next. Clearly, θ < [π/ 2, 3 π/ 2]. If
θ ∈ ( 0, π/ 2), then there exists a k ∈ { 2, 3, . . .} such that π k+ 1 ≤ θ < π k . If we choose n = k + 1, we have

π ≤ n θ < ( k + 1) π
k

which is impossible since cos( n θ) ≥ 0. Similarly, we can derive a contradiction if we assume θ ∈ ( 3 π/ 2, 2 π).
Then 2 π − π/ k ≤ θ < 2 π − π/( k + 1) for some k ∈ { 2, 3, . . .}.

1.5 Lines and half planes in the complex plane


Exercise 1. Let C be the circle {z : |z − c| = r}, r > 0; let a = c + r cis α and put
{ z : Im ( z − a )
Lβ= = 0}
b

where b = cis β. Find necessary and su ffi cient conditions in terms of β that L β be tangent to C at a.

Solution. Not available.

7
1.6 The extended plane and its spherical representation
Exercise 1. Give the details in the derivation of (6.7) and (6.8).

Solution. Not available.

Exercise 2. For each of the following points in C, give the corresponding point of S : 0, 1 + i, 3 + 2 i.

Solution. We have (2x )


Φ( z) = .
| z| 2 + 1 , 2 |yz| 2 + 1 , | z|| 2z|−21+ 1

If z 1 = 0, then |z 1| = 0 and therefore


Φ( z 1) = ( 0, 0, − 1).

Thus, z 1 = 0 corresponds to the point ( 0, 0, − 1) on the sphere S . If z 2 = 1 + i, then |z 2|


= √ 2 and therefore
(2 )
Φ( z 2) = .
3 , 23 , 13

)
Thus, z 2 = 1 + i corresponds to the point ( 23 , 23 , 13 on the sphere S .

If z 3 = 3 + 2 i, then |z 3| = √ 13 and therefore


(3 )
Φ( z 3) = .
7 , 27 , 67

)
Thus, z 3 = 3 + 2 i corresponds to the point ( 3 7 , 2 7 , 67
on the sphere S .

Exercise 3. Which subsets of S correspond to the real and imaginary axes in C.

Solution. If z is on the real axes, then z = x which implies |z| 2 = x 2. Thus


( 2 x x 2 + 1 , 0, x 2 − 1 )
Φ( z) = .
x2+ 1

Therefore the set


{( 2 x ) }⊂S

|x∈R
x 2 + 1 , 0, x 2 x− 21+ 1

corresponds to the real axes in C. That means the unit circle x 2 + z 2 = 1 lying in the xz − plane corresponds to the real axes in C.

If z is on the imaginary axes, then z = iy which implies |z| 2 = y 2. Thus


( )
Φ( z) = 0, 2 y .
y 2 + 1 , y 2y−2 1+ 1

Therefore the set


{( ) }⊂S

0, 2 y |y∈R
y 2 + 1 , y 2y−2 1+ 1

corresponds to the imaginary axes in C. That means the unit circle y 2 + z 2 = 1 lying in the yz − plane corresponds to the imaginary axes in C.

8
Exercise 4. Let Λ be a circle lying in S . Then there is a unique plane P in R 3 such that P ∩ S = Λ. Recall from analytic geometry that

P = {(x 1, x 2, x 3) : x 1 β 1 + x 2 β 2 + x 3 β 3 = l}

where ( β 1, β 2, β 3) is a vector orthogonal to P and l is some real number. It can be assumed that β 21+ β 22+ β 23 =
1. Use this information to show that if Λ contains the point N then its projection on C is a straight line. Otherwise, Λ projects onto a
circle in C.

Solution. Not available.

Exercise 5. Let Z and Z ′ be points on S corresponding to z and z ′ respectively. Let W be the point on S corresponding to z + z ′. Find the
coordinates of W in terms of the coordinates of Z and Z ′.

Solution. Not available.

9
Chapter 2

Metric Spaces and the Topology of C

2.1 Definitions and examples of metric spaces


Exercise 1. Show that each of the examples of metric spaces given in (1.2)-(1.6) is, indeed, a metric space. Example (1.6) is the only
one likely to give any di ffi culty. Also, describe B(x; r) for each of these examples.

Solution. Not available.

Exercise 2. Which of the following subsets of C are open and which are closed: (a) {z : |z| < 1}; ( b) the real axis; (c) {z : z n = 1 for some
integer n ≥ 1}; ( d) {z ∈ C is real and 0 ≤ z < 1}; ( e) {z ∈ C : z is real and
0 ≤ z ≤ 1}?

Solution. We have (a) A := {z ∈ C : | z|

< 1}

Let z ∈ A and set ε z = 1 −| z| 2 , then B(z, ε z) ⊂ A is open and therefore A = ⋃ z ∈ A B(z, ε z) is open also. A
cannot be closed, otherwise A and C − A were both closed and open sets yet C is connected. (b) B := {z ∈ C : z = x + iy, y

= 0} ( the real axis)

Let z ∈ C − B, then Im z , 0. Set ε z = | Im z| 2, then B(z, ε z) ⊂ C − B.Hence B is closed since its


complement is open. For any real x and any ε > 0 the
point x + i ε
2∈ B(x, ε) but x + i ε 2∈ C − R. Thus B is not open.

(c) C := {z ∈ C : z n = 1 for some integer n ≥ 1}

Claim: C is neither closed nor open. C is not open because if z n = 1 then z = r cis( θ) with r = 1 and any ε- ball around z would
contain an element z ′ := ( 1 + ε

4) cis( θ).
To show that C cannot be closed, note that for p q∈ Q with p ∈ Z and q ∈ N the number z :=
cis ( p q 2 π) ∈ C since
z q = cis( p 2 π) = cos( p 2 π) + i sin( p 2 π) = 1.

Now fix x ∈ R − Q and let {x n}n be a rational sequence that converges to x. Let m be any natural number. Now z m = 1 implies that sin(
mx 2 π) = 0 which in turn means that mx ∈ Z contradicting the choice x ∈ R − Q. We have constructed a sequence of elements of
C that converges to a point that is not element of C. Hence C is not closed.

10
(d) D := {z ∈ C : z is real and 0 ≤ z < 1}

The set cannot be open by the argument given in 2.1 ) and it is not closed because z n := 1 − 1 n is a sequence in D that converges

to a point outside of D. (e) E := {z ∈ C : z is real and 0 ≤ z ≤ 1}

This set E is not open by the observation in 2.1 ) and it is closed because its complement is open: If
z , E and z real, then B(z, min{| x|,|x − 1|} 2
) is contained in the complement of E, if z is imaginary then
B(z, | Im y|
2) is completely contained in the complement of E.

Exercise 3. If (X, d) is any metric space show that every open ball is, in fact, an open set. Also, show that every closed ball is a closed
set.

Solution. Not available.

Exercise 4. Give the details of the proof of (1.9c).

Solution. Not available.

Exercise 5. Prove Proposition 1.11.

Solution. Not available.

Exercise 6. Prove that a set G ⊂ X is open if and only if X − G is closed.

Solution. Not available.

Exercise 7. Show that ( C ∞, d) where d is given (I. 6.7) and (I. 6.8) is a metric space.

2| z − z ′ | 2 (1+| z| 2)1/2 , z ∈ C is a
Solution. To show ( C ∞, d) with d(z, z ′) =
[(1+| z| 2)( 1+| z ′ | 2)]1/2 for z, z ′ ∈ C and d(z, ∞) =
metric space.
i) d(z, z ′) ≥ 0.
2| z − z ′ |
Since |z − z ′| ≥ 0, we have d(z, z ′) = [(1+| z| 2)( 1+| z ′ | 2)]1/2 ≥ 0 for all z, z ′ ∈ C ( the denominator is always positive).
2 (1+| z| 2)1/2 ≥ 0 for all z ∈ C.
Obviously, d(z, ∞) =
ii) d(z, z ′) = 0 i ff z = z ′.
2| z − z ′ |
We have 2| z − z ′| = 0 i ff z = z ′. Therefore, d(z, z ′) = [(1+| z| 2)( 1+| z ′ | 2)]1/2 = 0 i ff z = z ′. d( ∞, ∞) =
2 (1+| z| 2)1/2 = 0. Thus, d( ∞, 0) = 0 i ff z = ∞.
lim z →∞
iii) d(z, z ′) = d(z ′, z). We
2| z − z ′ | 2| z ′ − z| 2 (1+| z| 2)1/2 =
have d(z, z ′) = [(1+| z| 2)( 1+| z ′ | 2)]1/2 = [(1+| z ′ | 2)( 1+| z| 2)]1/2 = d(z ′, z) for all z, z ′ ∈ C. Also d(z, ∞) =
d( ∞, z) by the symmetry of d(z, z ′) in general.
iv) d(z, z ′) ≤ d(z, x) + d(x, z ′).
We have

2| z − z ′|
d(z, z ′) =
[(1 + | z| 2)( 1 + | z ′| 2)]1/2
2| z − x| 2| x − z ′|

[(1 + | z| 2)( 1 + | x| 2)]1/2 + [(1 + | x| 2)( 1 + | z ′| 2)]1/2
= d(z, x) + d(x, z ′)

for all x, z, z ′ ∈ C. The first inequality will be shown next. First, we need
the following equation

( z − z ′)( 1 + ¯ xx) = (z − x)( 1 + ¯ xz ′) + ( x − z ′)( 1 + ¯ xz), (2.1)

11
which follows from a simple computation

( z − x)( 1 + ¯ xz ′) + ( x − z ′)( 1 + ¯ xz) = z − x + ¯ xz ′ z + x ¯ xz ′ + x − z ′ + x ¯ xz − ¯ xz ′ z

=z+x¯ xz − z ′ − x ¯ xz ′
= ( z − z ′)( 1 + ¯ xx).

Using ( 2.1 ) and taking norms gives

|( z − z ′)( 1 + ¯ xx)| = |(z − z ′)( 1 + | x| 2)| = |( z − z ′)|( 1 + | x| 2)

= |( z − x)( 1 + ¯ xz ′) + ( x − z ′)( 1 + ¯ xz)|

≤ | z − x|( 1 + | x| 2)1/2( 1 + | z ′| 2)1/2 + | x − z ′|( 1 + | x| 2)1/2( 1 + | z| 2)1/2 (2.2)

where the last inequality follows by

| z + ¯xz ′| ≤ ( 1 + | x| 2)1/2( 1 + | z ′| 2)1/2

⇔ (1 + ¯ xz ′)( 1 + x ¯ z ′) ≤ ( 1 + | x| 2)( 1 + | z ′| 2)

and

| z + ¯xz| ≤ ( 1 + | x| 2)1/2( 1 + | z| 2)1/2

⇔ (1 + ¯ xz)( 1 + x ¯ z) ≤ ( 1 + | x| 2)( 1 + | z| 2)

since

(1 + ¯ xz)( 1 + x ¯ z) ≤ ( 1 + | x| 2)( 1 + | z| 2)

⇔ x̄z + x ¯ z ≤ | x| 2 + | y| 2
xz

⇔ 2 Re(x ¯z) ≤ | x| 2 + | y| 2

which is true by Exercise 4 part 2 on page 3. Thus, dividing ( 2.2 ) by ( 1 + | x| 2)1/2 yields

| z − z ′|( 1 + | x| 2)1/2 ≤ | z − x|( 1 + | z ′| 2)1/2 + | x − z ′|( 1 + | z| 2)1/2.

Multiplying this by
2
(1 + | x| 2)1/2( 1 + | z ′| 2)1/2( 1 + | z| 2)1/2
gives the assertion above.

Exercise 8. Let (X, d) be a metric space and Y ⊂ X. Suppose G ⊂ X is open; show that G ∩ Y is open in
( Y, d). Conversely, show that if G 1 ⊂ Y is open in (Y, d), there is an open set G ⊂ X such that G 1 = G ∩ Y.

Solution. Set G 1 = G ∩ Y, let G be open in (X, d) and Y ⊂ X. A fim de mostrar que G 1 é aberta em (Y, d), escolher um ponto arbitrário p ∈ G 1. então p ∈
G e L uma vez que é aberto, existe um > 0 de tal modo que

B X ( p; ) ⊂ G.

Mas então
B Y ( p; ) = B X ( p; ) ∩ Y ⊂ G ∩ Y = L 1

o que prova que o símbolo p representa um ponto interior de L 1 na métrica d. assim L 1 é aberto em Y (proposição 1.13a).

Seja G 1 ser um conjunto aberto em Y. Então, para cada p ∈ G 1, existe uma - bola

B Y ( p: ) ⊂ G 1.

12
portanto ⋃
G1= B Y ( p; ).
p ∈ G1

Desde podemos escrever

B Y ( p; ) = B X ( p; ) ∩ Y,

Nós temos
⋃ ⋃ ( B X ( p; ) ∩ Y) ⋃
G1= B Y ( p; ) = = B X ( p; ) ∩ Y = L ∩ Y
p ∈ G1 p ∈ G1 p ∈ G1

onde L = ⋃ p ∈ G1B X ( p; ) É aberta em (X, d). (1.9c proposição uma vez que cada B X ( p; ) está aberto).

Exercício 9. Fazer Exercício 8 com “fechado” no lugar de “aberto”.

Solução. Não disponível.

Exercício 10. Prove Proposição 1.13.

Solução. Não disponível.

Exercício 11. Mostram que { cis k: ka número inteiro não negativo} é densa em T = {z ∈ C: | z | = 1}. Para que os valores de θ é { cis ( k θ): ka inteiro não
negativo} denso em T?

Solução. Não disponível.

2.2 Connectedness
Exercício 1. O objectivo deste exercício é mostrar que um subconjunto ligado de R é um intervalo. (A) Mostre que um conjunto A ⊂ R é um i

intervalo ff para quaisquer dois pontos a e b em A com um < b, o intervalo


[ a, b] ⊂ UMA.

(B) Usar a parte (a) para mostrar que se um conjunto A ⊂ R está ligado, em seguida, é um intervalo.

Solução. Não disponível.

Exercício 2. Mostram que os conjuntos S e T na prova do Teorema 2.3 estão abertas.

Solução. Não disponível.

Exercício 3. Qual dos seguintes subconjuntos X de C estão conectados; se X não está conectado, quais são os seus componentes: (a) X = {z: |
z | ≤ 1} ∪ { z: | z - 2 | <1}. ( b) X = [ 0, 1) ∩ { 1 + 1 n: n ≥ 1}. ( c) X = C - ( UMA ∩ B) em que A = [ 0, ∞) e B = {z = r cis θ: r = θ, 0 ≤ θ ≤ ∞}?

Solução. a) De fi ne X = {z: | z | ≤ 1} ∪ { z: | z - 2 | <1}: = UMA ∪ B. Ele su ffi ces para mostrar que X é conectado-caminho. Obviamente A é
conectado-caminho e B é conectado-caminho. Em seguida, vamos mostrar que X é conectado-path. Recorde-se que um espaço é caminho-se
ligado por quaisquer dois pontos x e y existe uma função f contínuo a partir do intervalo [ 0, 1] a X com f ( 0) = x e f ( 1) = y (esta função f é chamado
o caminho de x para y). deixe-x ∈ A e y ∈ fi B e de ne a função f (t): [ 0, 1] → X por


•• (1 - 3 t) x + 3 tRe (x),
•• 0 ≤ t ≤ 13
••
f (t) =
13 < t≤2
•• (2 - 3 t) Re (x) + ( 3 t - 1) Re (y),
•• 3
•• 23 < t ≤ 1.
(3 - 3 t) Re (y) + ( 3 t - 2) y,

13
Esta função é, obviamente, contínua, uma vez que f ( 1/3) = lim t → 1 - f (t) = lim t → 13 + f (t) = Re (x) ∈ x e
3

f ( 2/3) = lim t → 23 - f (t) = lim t → 2 + f (t) = Re (y) ∈ X. Além disso, temos f ( 0) = x e f ( 1) = y. Assim sendo
3

X é conectado-caminho e, consequentemente, X está ligado.

b) Não há nenhuma maneira de se conectar { 2} e { 3


2}. Portanto X não está ligado. Os componentes são [ 0, 1), {2},
{ 3 2}, {43}, . . ., { 1 + 1 N}.
c) X não está ligado, já que não há maneira de se conectar ( 2, 1) e ( 1, - 2). O K - th componente é dada pela
C - { UMA ∩ B} onde
A = [ 2 π k - 2 π, 2 π k)

e
B = {z = r cis θ: r = θ, 2 π k - 2 π ≤ θ < 2 π k}, k ∈ { 1, 2, 3, . . .}.

Exercício 4. Prove o seguinte generalização do Lema 2.6. Se {D j: j ∈ J} é um conjunto de subconjuntos ligados de X e, se para cada j e
k em J que tem D j ∩ D k, em seguida, D = ⋃ { D j: j ∈ J} é ligado.

Solução. Seja D = ⋃ j ∈ J D j e C = { D j: j ∈ J}. Se D está ligada, poderíamos escrever D como a união disjunta

UMA ∪ B em que A e B são subconjuntos não vazios de X. Assim, para cada um C ∈ C quer C ⊂ A ou C ⊂ B. Temos C ⊂ UMA ∀ C ∈ C ou C ⊂ B ∀
C ∈ C. Se não, então existem E, F ∈ C tal que E ⊂ A e F ⊂ B. Mas, assumimos A ∪ B é separado e, assim, E ∪ F é separado o que contraria a
hipótese de E ∩ F, ∅.
Portanto, todos os membros da C estão contidos em qualquer uma ou todas B. Assim, tanto D = A e B = ∅ ou D = B e A = ∅. Ambos contradizendo o fato de
que A, B são considerados não vazio. Conseqüentemente,


D= Dj
j∈J

está conectado.

Exercício 5. Mostrar que, se F ⊂ X é fechada e, em seguida, ligado para cada par de pontos a, b e F em cada
> 0 existem pontos z 0, z 1, . . . , z n em F com z 0 = A, Z n = b e d (z k - 1, z k) < para 1 ≤ k ≤ n. É o
hipótese de que F ser fechada necessário? Se F é um conjunto que satisfaz esta propriedade, em seguida, F não é necessariamente ligada, mesmo que F
está fechada. Dê um exemplo para ilustrar isso.

Solução. Não disponível.

2.3 integralidade e Sequências


Exercício 1. Provar proposição 3,4.

Solução. a) Um conjunto é i fechado ff ele contém todos os seus pontos-limite. deixe-S ⊂ X ser um conjunto. “ ⇐ “: Suponha S contém todos os seus
pontos-limite. Temos que mostrar que S é fechado ou que S c está aberto. deixe-x ∈ S c. Por hipótese x não é um ponto limite e, portanto, existe uma
aberto - bola em torno x, B (x; ) De tal modo que B (x; ) ∩ S = ∅ ( negação de 1.13f proposi). Então B (x; ) ⊂ S c e, por conseguinte, S c está aberto. “ ⇒ “:
Let S ser fechada e x ser um ponto limite. Nós reivindicamos x ∈ S. Se não, S c ( abrir) seria uma vizinhança aberta de x, que não se cruzam S.
(Proposição 1.13f de novo) o que contraria o facto de que x é um ponto limite de S.

b) Se A ⊂ X, então A - = UMA ∪ { x: x é um ponto limite de A}: = A ∪ UMA '.


Seja A ⊂ X ser um conjunto. “ ⊆ “:
Deixe-x ∈ ¯ A. Queremos mostrar x ∈ UMA ∪ UMA '. Se x ∈ A, então, obviamente x ∈ UMA ∪ UMA '. Suponha que x < A. Desde

X∈¯ A, temos (Proposição 1.13f) que para cada > 0, B (x; ) ∩ UMA , ∅. Porque x < A, B (x; ) devo

14
Um intersectam num ponto di ff erent de x. Em particular, para qualquer número inteiro n existe um ponto x n em B (x; 1 n) ∩ A. Assim, d (x, x n) < 1

n o que implica x n → x (ver prova de proposição 3,2). então x ∈ UMA ', assim x ∈ UMA ∪ UMA '.

“ ⊇ “: Para mostrar um ∪ UMA ' ⊆ UMA -. deixe-x ∈ UMA ∪ UMA '. Se x ∈ Um, então x ∈ UMA - ( UMA ⊂ UMA -). Agora, suponha x ∈ UMA ' mas não em A. Então existe {x n} ⊂ A
com lim n → ∞ X n = x. Segue-se, ∀ > 0 B (x; ) ∩ UMA , ∅ desde {x n} ⊂ A. Por 1.13f Proposição obtemos x ∈ UMA -.

Exercício 2. Fornecer os detalhes da prova da Proposição 3.8.

Solução. Não disponível.

Exercício 3. Mostram que diam A = diam A -.

Solução. Não disponível.

Exercício 4. deixe-z N, z ser em pontos C e deixar que estaria a métrica em C ∞. Mostre que | z n - z | → 0 Se e apenas se d (z N, z) → 0. Também mostram que, se | z n
| → ∞ em seguida, {z n} é Cauchy em C ∞. ( Deve {z n} convergem C ∞?)

Solução. Primeiro assumir que | z n - z | → 0, então

2
d (z N, z) = √ ( 1 + | z n | 2) ( 1 + | z | 2) | z n - z | → 0

uma vez que o denominador √ ( 1 + | z n | 2) ( 1 + | z | 2) ≥ 1 é delimitada abaixo de distância a partir de 0. Para ver o inverso, deixe d (z N, z) → 0 ou
equivalentemente d 2 ( z N, z) → 0. Precisamos mostrar que, se z n → z d-norma, então | z n | 9 ∞
porque caso contrário o denominador cresce sem limites. Na verdade, vamos mostrar que, se | z n | → ∞, em seguida d (z N, z) 9 0 para qualquer z ∈ C. Então

4 (| z n | 2 - z n ¯ z - ¯ z n z + | z | 2)
d 2 ( z N, z) = ( 1 + | z n | 2) ( 1 + | z | 2)

4 (| z n | 2 - 2 Re (z n ¯ z) + | Z | 2)
= ( 1 + | z | 2) | z n | 2 + 1 + | z | 2

)
z)
4 (1 - 2 Re (z n ¯
| zn|2+| z |2 | zn|2
= se | z n | , 0
1 + | z | 2 + 1 + | z |2
| zn|2

nomeadamente se | z n | → ∞, d (z N, z) → 4
1+ | z | 2, 0. Isto mostra que o denominador permanece como delimitada
d (z N, z) → 0 e, por conseguinte, o numerador 2 | z n - z | → 0. Daí a convergência na d-norma implica convergência de | · | - norma para números Z N, z ∈ C. Em
seguida assumir que | z n | → ∞. Então é claro que também √ 1 + | z n | 2 → ∞ e, por conseguinte, d (z N, ∞) =
√ 1+ 2| z n | 2 → 0. A última coisa a mostrar é que, se z n → ∞ em (C ∞, d), também | z n | → ∞. Mas

d (z N, ∞) → 0 implica √ 1 + | z n | 2 → ∞ o qual é equivalente a | z n | → ∞.

Exercício 5. Mostram que cada seqüência convergente em (X, d) é uma sequência de Cauchy.

Solução. Seja {x n} ser uma sequência convergente com limite de x. Isto é, dado > 0 ∃ N tal que d (x N, x) < 2 se n> N e d (x, x m) < 2 Se m> N. Assim,

d (x N, X m) ≤ d (x N, x) + D (x, x m) < 2 + 2 = , ∀ n, m ≥ N

e, por conseguinte, {x n} é uma sequência de Cauchy.

15
Exercício 6. Dê três exemplos de espaços métricos não completos.
√∫ 1
Solução. Exemplo 1: Deixa X = C [ - 1, 1] e a métrica d ( f, g) =
-1[ f (x) - g (x)] 2 dx, f, g ∈ X. Considere
a sequência de Cauchy

•• 0, -1≤X≤0
••
••
f n ( x) = •• nx, 0 < X ≤ 1 n .
••
•• 1n< X ≤ 1
1,

É óbvio que a função limite de f é descontínua. Assim, o espaço métrico (X, d) não é completa. Exemplo 2: Deixa X = ( 0, 1] com métrica d
(x, y) = | x - y |, x, y ∈ X. A sequência { 1 n} é Cauchy, mas converge para 0, que não está no espaço. Assim, o espaço métrico não está
completa. Exemplo 3: Deixa X = Q, os racionais, com métrica d (x, y) = | x - y |, x, y ∈ X. A sequência definida por X 1 = 1,

X n + 1 = X n 2 + 1 X n is a Cauchy sequence of rational numbers. The limit √ 2 is not a rational number. Therefore,
the metric space is not complete.

Exercise 7. Put a metric d on R such that |x n − x| → 0 if and only if d(x n, x) → 0, but that {x n} is a Cauchy sequence in ( R, d) when |x n| → ∞. ( Hint:
Take inspiration from C ∞.)

Solution. Not available.

Exercise 8. Suppose {x n} is a Cauchy sequence and {x n k } is a subsequence that is convergent. Show that
{ x n} must be convergent.

Solution. Since {x n k } is convergent, there is a x such that x n k → x as k → ∞. We have to show that x n → x as n → x. Let > 0. Then we have ∃ N
∈ N such that d(x n, x m) < 2 ∀ n, m ≥ N since {x n} is Cauchy and
∃ M ∈ N such that d(x n k , x) < 2 ∀ n k ≥ M since x n k → x as k → ∞. Now, fix n k 0 > M + N, then

d(x n, x) ≤ d(x n, x n k 0 ) + d(x n k 0 , x) < 2 + 2 = , ∀ n ≥ N.

Thus, x n → x as n → ∞ and therefore {x n} is convergent.

2.4 Compactness
Exercise 1. Finish the proof of Proposition 4.4.

Solution. Not available.

Exercise 2. Let p = (p 1, . . . , p n) and q = (q 1, . . . , q n) be points in R n with p k < q k for each k. Let R = [p 1, q 1] × . . . × [ p n, q n] and show that

• n • 12

•• ∑ ••
•• ••
diam R = d(p, q) = •• ( q k − p k) 2 •• .
k= 1

Solution. By definition
diam(R) = sup d(x, y).
x ∈ R,y ∈ R

Obviously, R is compact, so we have

diam(R) = max
x ∈ R,y ∈ R d(x, y).

16
Let x = (x 1, . . . , x n) ∈ R and y = (y 1, . . . , y n) ∈ R. Then clearly, p i ≤ x i ≤ q i and p i ≤ y i ≤ q i for all i = 1, . . . , n. We also have

( y i − x i) 2 ≤ ( q i − p i) 2, ∀ i = 1, . . . , n. (2.3)

Therefore, by ( 2.3 ) we obtain


√√ n √√ n
∑ ∑
d(x, y) = ( y i − x i) 2 ≤ ( q i − p i) 2 = d(p, q).
i= 1 i= 1

Note that we get equality if for example x i = p i and y i = q i for all i = 1, . . . , n. Thus the maximum distance is obtained, so

√√ n

diam(R) = sup d(x, y) = ( q i − p i) 2 = d(p, q).
x ∈ R,y ∈ R
i= 1

Exercise 3. Let F = [a 1, b 1] × . . . × [ a n, b n] ⊂ R n and let > 0; use Exercise 2 to show that there are rectangles R 1, . . . , R m such that F = ⋃ mk= 1 R k and
diam R k < for each k. If x k ∈ R k then it follows that R k ⊂ B(x k; ).

Solution. Not available.

Exercise 4. Show that the union of a finite number of compact sets is compact.

Solution. Let K = ⋃ ni= 1 K i be a finite union of compact sets. Let {G λ}λ ∈ Γ be an open cover of K, that is

K⊂⋃ G λ.
λ∈Γ

Of course, {G λ}λ ∈ Γ is an open cover of each K i, i = 1, . . . , n, that is

Ki⊂ ⋃ G λ, ∀ i = 1, . . . , n.
λ∈Γ

Since each K i is compact,



K i ⊂ ki G i, j
j= 1

for each i = 1, . . . , n. Thus


⋃n ⋃ ⋃k i
K= Ki⊂ n G i, j
j= 1 i= 1 j= 1

which is a finite union and therefore K is compact.

Exercise 5. Let X be the set of all bounded sequences of complex numbers. That is, {x n} ∈ X i ff sup{| x n| :
n ≥ 1} < ∞. If x = {x n} and y = {y n}, define d(x, y) = sup{| x n − y n| : n ≥ 1}. Show that for each x in X and
> 0, ¯ B(x; ) is not totally bounded although it is complete. (Hint: you might have an easier time of it if you first show that you can
assume x = ( 0, 0, . . .).)

Solution. Not available.

Exercise 6. Show that the closure of a totally bounded set is totally bounded.

17
Solution. Let (X, d) be a given metric space and let S ⊂ X be totally bounded. Let > 0. Since S is totally bounded, we have by Theorem
4.9 d) p. 22 that there exist a finite number of points x 1, . . . , x n ∈ S such that


S⊆n B(x k; / 2).
k= 1

Taking the closure, gives the desired result


S¯ ⊆ n B(x k; ).
k= 1

2.5 Continuity
Exercise 1. Prove Proposition 5.2.

Solution. Not available.

Exercise 2. Show that if f and g are uniformly continuous (Lipschitz) functions from X into C then so is f + g.

Solution. The distance in C is ρ( x, y) = |x − y|. The distance in X is d(x, y). Let f : X → C be Lipschitz, that is, there is a constant M > 0 such
that

ρ( f (x), f (y)) = | f (x) − f (y)| ≤ Md(x, y), ∀ x, y ∈ X

and g : X → C be Lipschitz, that is, there is a constant N > 0 such that

ρ( g(x), g(y)) = |g(x) − g(y)| ≤ Nd(x, y), ∀ x, y ∈ X.

We have

ρ( f (x) + g(x), f (y) + g(y)) = | f (x) + g(x) − f (y) − g(y)| = | f (x) − f (y) + g(x) − g(y)|

≤ | f (x) − f (y)| + |g(x) − g(y)|

≤ Md(x, y) + Nd(x, y) = (M + N)d(x, y), ∀ x, y ∈ X.

Since there is a constant K = M + N > 0 such that

ρ( f (x) + g(x), f (y) + g(y)) ≤ Kd(x, y), ∀ x, y ∈ X,

we have that f + g is Lipschitz.

Now, let f , g be both uniformly continuous, that is,

∀ 1 > 0 ∃ δ 1 > 0 such that ρ( f (x), f (y)) = | f (x) − f (y)| < 1 whenever d(x, y) < δ 1

and
∀ 2 > 0 ∃ δ 2 > 0 such that ρ( f (x), f (y)) = | f (x) − f (y)| < 2 whenever d(x, y) < δ 2.

We have

ρ( f (x) + g(x), f (y) + g(y)) = | f (x) + g(x) − f (y) − g(y)| = | f (x) − f (y) + g(x) − g(y)|

≤ | f (x) − f (y)| + |g(x) − g(y)|

≤ 1 + 2,

18
whenever d(x, y) < δ 1 and d(x, y) < δ 2, that is, whenever d(x, y) < min( δ 1, δ 2). So, choosing = 1 + 2 and
δ = min( δ 1, δ 2), we have shown that

∀ > 0 ∃ δ > 0 such that ρ( f (x), f (y)) = | f (x) − f (y)| < whenever d(x, y) < δ.

Thus f + g is uniformly continuous.

Exercise 3. We say that f : X → C is bounded if there is a constant M > 0 with | f (x)| ≤ M for all x in X. Show that if f and g are bounded
uniformly continuous (Lipschitz) functions from X into C then so is f g.

Solution. Let f be bounded, that is, there exists a constant M 1 > 0 with | f (x)| < M 1 for all x ∈ X and let g be bounded, that is, there exists a
constant M 2 > 0 with |g(x)| < M 2 for all x ∈ X. Obviously f g is bounded, because

| f (x)g(x)| ≤ | f (x)| |g(x)| ≤ M 1 · M 2 ∀ x ∈ X.

So, there exists a constant M = M 1 M 2 with

| f (x)g(x)| ≤ M ∀ x ∈ X.

Let f be Lipschitz, that is, there exists a constant N 1 > 0 such that

ρ( f (x), f (y)) ≤ N 1 d(x, y) ∀ x, y ∈ X

and let g be Lipschitz, that is, there exists a constant N 2 > 0 such that

ρ( g(x), g(y)) ≤ N 1 d(x, y) ∀ x, y ∈ X.

Now,

ρ( f (x)g(x), f (y)g(y)) = | f (x)g(x) − f (y)g(y)| =

= | f (x)g(x) − f (x)g(y) + f (x)g(y) − f (y)g(y)|

≤ | f (x)g(x) − f (x)g(y)| + | f (x)g(y) − f (y)g(y)|

≤ | f (x)| |g(x) − g(y)| + | f (x)| |g(y) − g(y)|

≤ M 1 N 2 d(x, y) + M 2 N 1 d(x, y) = (M 1 N 2 + M 2 N 1) d(x, y) ∀ x, y ∈ X.

Thus, there exists a constant K = M 1 N 2 + M 2 N 1 > 0 with

ρ( f (x)g(x), f (y)g(y)) ≤ Kd(x, y) ∀ x, y ∈ X,

so f g is Lipschitz and bounded. It follows immediately that f g is uniformly continuous (see page 25 after Definition 5.6). For the sake
of completeness, we give the proof.

Let f and g be both bounded and uniformly continuous, that is

∃ M 1 such that | f (x)| ≤ M 1 ∀x∈X

∃ M 2 such that |g(x)| ≤ M 2 ∀x∈X

∀ 1 > 0 ∃ δ 1 > 0 such that ρ( f (x), f (y)) = | f (x) − f (y)| < 1 whenever d(x, y) < δ 1

∀ 2 > 0 ∃ δ 2 > 0 such that ρ( f (x), f (y)) = | f (x) − f (y)| < 2 whenever d(x, y) < δ 2.

19
It remains to verify that f g is uniformly continuous, since we have already shown that f g is bounded. We have

ρ( f (x)g(x), f (y)g(y)) = | f (x)g(x) − f (y)g(y)| =

= | f (x)g(x) − f (x)g(y) + f (x)g(y) − f (y)g(y)|

≤ | f (x)g(x) − f (x)g(y)| + | f (x)g(y) − f (y)g(y)|

≤ | f (x)| |g(x) − g(y)| + | f (x)| |g(y) − g(y)|

≤ M 1 2 + M 2 1,

whenever d(x, y) < min( δ 1, δ 2). So choosing = M 1 2 + M 2 1 and δ = min( δ 1, δ 2), we have ∀ > 0 ∃ δ > 0
such that
| f (x)g(x) − f (y)g(y)| <

whenever d(x, y) < δ. Thus, f g is uniformly continuous and bounded.

Exercise 4. Is the composition of two uniformly continuous (Lipschitz) functions again uniformly continuous (Lipschitz)?

Solution. Not available.

Exercise 5. Suppose f : X → Ω is uniformly continuous; show that if {x n} is a Cauchy sequence in X then


{ f (x n)} is a Cauchy sequence in Ω. Is this still true if we only assume that f is continuous? (Prove or give a counterexample.)

Solution. Assume f : X → Ω is uniformly continuous, that is, for every > 0 there exists δ > 0 such that
ρ( f (x), f (y)) < if d(x, y) < δ. If {x n} is a Cauchy sequence in X, we have, for every 1 > 0 there exists N ∈ N such that d(x n, x m) < 1 for all n, m ≥ N.
But then, by the uniform continuity, we have that

ρ( f (x n), f (x m)) < ∀ n, m ≥ N

whenever d(x n, x m) < δ which tells us that { f (x n)} is a Cauchy sequence in Ω.


If f is continuous, the statement is not true. Here is a counterexample: Let f (x) = 1 x which is continuous on ( 0, 1). The sequence x n = 1 n is
apparently convergent and therefore a Cauchy sequence in X. But
{ f (x n)} = { f ( 1 n )} = { n} is obviously not Cauchy. Note that f (x) = 1 x is not uniformly continuous on ( 0, 1).
To see that pick = 1. Then there is no δ > 0 such that | f (x) − f (y)| < 1 whenever |x − y| < δ. Assume there exists such a δ. WLOG assume δ < 1 since
the interval ( 0, 1) is considered. Let y = x+ δ/ 2 and set x = δ/ 2,
then ∣∣∣∣∣ 1 ∣∣∣∣∣ = y − x

| f (x) − f (y)| =
x−1 y xy = δ/ 2 δ/ 2 · δ = 1 δ > 1,

that is no matter what δ < 1 we choose, we always obtain | f (x) − f (y)| > 1. Therefore f (x) = 1 x cannot be uniformly continuous.

Exercise 6. Recall the definition of a dense set (1.14). Suppose that Ω is a complete metric space and that f : (D, d) → ( Ω; ρ) is uniformly
continuous, where D is dense in (X, d). Use Exercise 5 to show that there is a uniformly continuous function g : X → Ω with g(x) = f (x)
for every x in D.

Solution. Not available.

Exercise 7. Let G be an open subset of C and let P be a polygon in G from a to b. Use Theorems 5.15 and 5.17 to show that there is a
polygon Q ⊂ G from a to b which is composed of line segments which are parallel to either the real or imaginary axes.

20
Solution. Not available.

Exercise 8. Use Lebesgue’s Covering Lemma (4.8) to give another proof of Theorem 5.15.

Solution. Suppose f : X → Ω is continuous and X is compact. To show f is uniformly continuous. Let


> 0. Since f is continuous, we have for all x ∈ X there is a δ x > 0 such that ρ( f (x), f (y)) < / 2 whenever d(x, y) < δ x. In addition,


X= B(x; δ x)
x∈X

is an open cover of X. Since X is by assumption compact (it is also sequentially compact as stated in Theorem 4.9 p. 22), we can use
Lebesgue’s Covering Lemma 4.8 p. 21 to obtain a δ > 0 such that x ∈ X implies that B(x, δ) ⊂ B(z; δ z) for some z ∈ X. More precisely, x, y
∈ B(z; δ z) and therefore

ρ( f (x), f (z)) ≤ ρ( f (x), f (z)) + ρ( f (z), f (y)) < 2 + 2 =

and hence f is uniformly continuous on X.

Exercise 9. Prove the following converse to Exercise 2.5. Suppose (X, d) is a compact metric space having the property that for every > 0
and for any points a, b in X, there are points z 0, z 1, . . . , z n in X with z 0 = a, z n = b, and d(z k − l, z k) < for 1 ≤ k ≤ n. Then (X, d) is connected. (Hint:
Use Theorem 5.17.)

Solution. Not available.

Exercise 10. Let f and g be continuous functions from (X, d) to ( Ω, p) and let D be a dense subset of X. Prove that if f (x) = g(x) for x in D
then f = g. Use this to show that the function g obtained in Exercise 6 is unique.

Solution. Not available.

2.6 Uniform convergence


Exercise 1. Let { f n} be a sequence of uniformly continuous functions from (X, d) into ( Ω, ρ) and suppose that f = u − − lim f n exists. Prove
that f is uniformly continuous. If each f n is a Lipschitz function with constant M n and sup M n < ∞, show that f is a Lipschitz function. If sup M
n= ∞, show that f may fail to be Lipschitz.

Solution. Not available.

21
Chapter 3

Elementary Properties and Examples of Analytic


Functions

3.1 Power series


Exercise 1. Prove Proposition 1.5.

Solution. Not available.

Exercise 2. Give the details of the proof of Proposition 1.6.

Solution. Not available.

Exercise 3. Prove that limsup( a n+ b n) ≤ limsup a n+ limsup b n and lim inf( a n+ b n) ≥ lim inf a n+ lim inf b n
for bounded sequences of real numbers {a n} and {b n}.

Solution. Let r > limsup n →∞ a n ( we know there are only finitely many by definition) and let s > limsup n →∞
(same here, there are only finitely many by definition). Then r + s > a n + b n for all but finitely many n’s. This however, implies that

r + s ≥ limsup
n →∞ ( a n + b n).
Since this holds for any r > limsup n →∞ a n and s > limsup n →∞ b n, we have

limsup
n →∞ ( a n + b n) ≤ limsup n →∞ a n + limsup n →∞ b n.

Let r < lim inf n →∞ a n ( we know there are only finitely many by definition) and let s < lim inf n →∞ ( same here, there are only finitely many by
definition). Then r + s < a n + b n for all but finitely many n’s. This however, implies that

r + s ≤ lim inf
n →∞ ( a n + b n).
Since this holds for any r < lim inf n →∞ a n and s < lim inf n →∞ b n, we have

lim inf
n →∞ ( a n + b n) ≥ lim inf n →∞ a n + lim inf n →∞ b n.

Exercise 4. Show that lim inf a n ≤ limsup a n for any sequence in R.

22
Solution. Let m = lim inf n →∞ a n and b n = inf{ a n, a n+ 1, . . .}. Let M = limsup n →∞ a n. Take any s > M. Then, by definition of the limsup n →∞ a n = M, we
obtain that a n < s for infinitely many n’s which implies that b n < s for all n and hence limsup n →∞ b n = m < s. This holds for all s > M. But the
infimum of all these s’s is M. Therefore m ≤ M which is

lim inf
n →∞ a n ≤ limsup n →∞ a n.

Exercise 5. If {a n} is a convergent sequence in R and a = lim a n, show that a = lim inf a n = limsup a n.

Solution. Suppose that {a n} is a convergent sequence in R with limit a = lim n →∞ a n. Then by definition, we have: ∀ > 0 ∃ N > 0 such that ∀ n ≥ N,
we have |a n − a| ≤ , that is a − ≤ a n ≤ a + . This means that all but finitely many a n ’s are ≤ a + and ≥ a − . This shows that

a − ≤ lim inf ≤a+


n →∞ a n
︸ ︷︷ ︸
=: m

and
a − ≤ limsup ≤a+.
n →∞ a n
︸ ︷︷ ︸
=: M

By the previous Exercise 4, we also have

a−≤m≤M≤a+.

Hence,
0≤M−m≤2.

Since > 0 is arbitrary, we obtain m = M and further

a − ≤ lim inf
n →∞ a n ≤ limsup n →∞ a n ≤ a+,

we obtain
lim inf
n →∞ a n = limsup n →∞ a n = a.
Exercise 6. Find the radius of convergence for each of the following power series: (a) ∑ ∞ n= 0 a n z n, a ∈ C;
(b) ∑ ∞ n= 0 a n 2 z n, a ∈ C; ( c) ∑ ∞ n= 0 k n z n, k an integer , 0; ( d) ∑ ∞ n= 0 z n!.

Solution. a) We have ∑ ∞ n= 0 a n z n = ∑ ∞ k= 0 b k z n with b k = a k, a ∈ C. We also have,

limsup
k →∞ | b k| 1/ k = limsup k →∞ | a k| 1/ k = limsup k →∞ | a| = |a|.

Therefore, R = 1/| a|, so •


•• 1
• | a| , a,0
R = ••
• ∞, a = 0 .

b) In this case, b n = a n 2 where a ∈ C.


∣∣∣∣∣ b n ∣∣∣∣∣ = lim ∣∣∣∣∣∣ a n 2 ∣∣∣∣∣∣ = lim ∣∣∣∣∣∣ a n 2 ∣∣∣∣∣∣ = lim ∣∣∣∣∣ 1 ∣∣∣∣∣

R = lim
n →∞ b n+ 1 n →∞ a( n+ 1) 2 n →∞ a n 2+ 2 n+ 1 n →∞ a 2 n+ 1

•• 0, | a| > 1
••
1 ••
= lim •• 1, | a| = 1 .
n →∞ | a| 2 n+ 1 = ••
••
∞, | a| < 1

23
c) Now, b n = k n, k is an integer , 0. We have

R = limsup
n →∞ | b n| 1/ n = limsup n →∞ | k n| 1/ n = limsup n →∞ | k| = |k|.

So

•• 1k,
k > 0, k integer

R=1 ••
| k| = • − 1 k , k < 0, k integer .

d) We can write ∑ ∞ n= 0 z n! = ∑ ∞ k= 0 a k z k where



•• 0, k = 0
••
••
•• 2, k = 1

ak= ••
•• 1, k = n!, n ∈ N, n > 1
••
••
• 0, otherwise

Thus,
limsup
k →∞ | a k| 1/ k = limsup k →∞ | 1| 1/ k! = 1.
Therefore 1/ R = 1 which implies R = 1.

Exercise 7. Show that the radius of convergence of the power series

∑∞ ( − 1) n
n z n(n+ 1)
n= 1

is 1, and discuss convergence for z = 1, − 1, and i. (Hint: The nth coe ffi cient of this series is not ( − 1) n/ n.)

Solution. Rewrite the power series in standard form, then



∑∞ ∑∞ •• ( − 1) n
( − 1) n • if ∃ n ∈ N s.t. k = n(n + 1)
a k z k with a k = ••
n
.
n z n(n+ 1) = • 0 else
n= 1 n= 1

To find the radius of convergence we use the root criterion and therefore need the estimates
√n≤n √n
1 ≤ n(n+ 1) for n ∈ N.

The first inequality is immediate from the fact that n ≥ 1 and hence n 1 n(n+ 1) ≥ 1. For the second inequality
note that

n ≤ n n+ 1

⇔ n 1n(n+ 1) ≤ n 1n

√n≤n √n.
n(n+ 1)

Using this one obtains


√| ( − 1) n
n(n+ 1)

√n≤1
n|=1 n(n+ 1)

and

√| ( − 1) n
n(n+ 1)

√ n ≥ 1 √n n .
n|=1 n(n+ 1)

24
√ n → 1, thus 1 √ a n = 1, i.e. R = 1.
Vague memories of calculus classes tell me that n R= limsup n(n+ 1)

If z = 1 the series reduces to ∑ ∞ n= 1 ( − 1) n n which converges with the Leibniz Criterion.


If z = − 1 we note that the exponents n(n + 1) are always even integers and therefore the series is the same as in the previous case
of z = 1.
Now let z = i. The expression i n(n+ 1) will always be real, so if the series converges at z = i, it converges
to a real number. We also note that formally

∑∞ ∑∞ •• 1n
ifn mod 4 ∈ { 0, 1}
( − 1) n •
c n with ••
n i n(n+ 1) = • −1n if n mod 4 ∈ { 2, 3} .
n= 1 n= 1

Define the partial sums S k := ∑ kn= 0 c k. We claim that the following chain of inequalities holds

a) b) c) d)
0 ≤ S 4 k+ 3 < S 4k < S 4 k+ 4 < S 4 k+ 2 < S 4 k+ 1 ≤ 1.

To verify this, note that

16 k 2 + 8 k − 1 (4 k + 1)(4 k + 2)(4 k + 3) < 0, hence


S 4 k+ 3 − S 4 k = c 4 k+ 1 + c 4 k+ 2 + c 4 k+ 3 = − a)

S 4 k+ 4 − S 4 k = c 4 k+ 3 + c 4 k+ 4 = − 1
4 k + 3 + 1 4 k + 4 < 0, hence c)
S 4 k+ 2 − S 4 k+ 1 = c 4 k+ 2 < 0, hence d).

Relation b) is obvious and so are the upper bound c 1 = 1 and the non-negativity constraint. We remark that
{ S 4 k+l}k ≥ 1, l ∈ { 0, 1, 2, 3} describe bounded and monotone subsequences that converge to some point. Now that |c n| ↘ 0 the di ff erence
between S 4 k+l and S 4 k+m, l, m ∈ 0, 1, 2, 3 tends to zero, i.e. all subsequences converge to the same limit. Therefore the power series
converges also in the case of z = i.

3.2 Analytic functions


Exercise 1. Show that f (z) = |z| 2 = x 2 + y 2 has a derivative only at the origin.

Solution. The derivative of f at z is given by

f (z + h) − f (z)
f ′( z) = lim , h∈C
h→0 h

provided the limit exist. We have

f (z + h) − f (z) z + ¯h) − z ¯ z z + h ¯z + z ¯h + h ¯h − z ¯ z
= | z + h| 2 − | z| 2 = ( z + h)( ¯ =z¯
h h h h

h h =: D.
=¯ z + ¯h + z ¯

If the limit of D exists, it may be found by letting the point h = (x, y) approach the origin (0,0) in the complex plane C in any manner.

1.) Take the path along the real axes, that is y = 0. Then ¯ h = h and thus

D = ¯ z + h + zh z+h+z
h=¯

25
and therefore, if the limit of D exists, its value has to be ¯ z + z.
2.) Take the path along the imaginary axes, that is x = 0. Then ¯ h = − h and thus

D = ¯ z − h − zh z−h−z
h=¯

and therefore, if the limit of D exists, its value has to be ¯ z − z.


Because of the uniqueness of the limit of D, we must have

zz̄ + z = ¯ z − z ⇐⇒ z = − z ⇐⇒ z = 0,

if the limit of D exists. It remains to show that the limit of D exists at z = 0. Since z = 0, we have that D = ¯ h
and thus the limit of D is 0. In summary, the function f (z) = |z| 2 = x 2 + y 2 has a derivative only at the origin with value 0.

Exercise 2. Prove that if b n, a n are real and positive and 0 < b = lim b n < ∞, a = limsup a n then ab = limsup( a n b n). Docs this remain true if the
requirement of positivity is dropped?

Solution. Let a = limsup


n →∞ a n < ∞. Then there exists a monotonic subsequence {a n k } of {a n} that converges
to a. Since lim
k →∞ b n k = b, lim n →∞ a n k b n k = ab. Hence, {a n k b n k } is a subsequence of a n b n that converges to ab. So
ab ≤ limsup
n →∞ a n b n. Hence, limsup n →∞ a n b n ≥ b limsup n →∞ a n.
Now, let a = limsup
n →∞ a n = ∞. Then there exists a subsequence {a n k } of {a n} such that lim k →∞ a n k = a > 0.

And, since lim


n →∞ b n > 0, lim k →∞ a n k b n k = ∞. Hence limsup n →∞ a n b n = ∞. In this second case, limsup n →∞ a n b n ≥
b limsup
n →∞ a n.
In both cases, we have established that ab ≤ limsup
n →∞ a n b n. Now, since for all n ∈ N, a n > 0 and b n > 0,
1
consider lim =1
b n and a n replaced
n →∞ bn b . Applying the inequality we have established replacing b n with 1
with a n b n:

1
limsup ( a n b n) ≥ 1
n →∞ a n = limsup n →∞ bn b limsup
n →∞ a n b n.

Rearranging,

limsup
n →∞ a n b n ≤ b limsup n →∞ a n.

It follows that ab = limsup


n →∞ a n b n as required.

Now, consider the case where we drop the positivity requirement. Let b n = 0, − 12 , 0, − 1 3, 0, − 14 , . . . and
note 0 = b = lim
n →∞ b n < ∞. Also let a n = 0, − 2, 0, − 3, 0, − 4 . . . and note limsup n →∞ a n = a = 0. In this case,
ab = 0 , 1 = limsup
n →∞ a n b n.

Exercise 3. Show that lim n 1/ n = 1.

26
Solution. Let n ∈ N. Also let a = n 1/ n. Then

a = n 1/ n

⇐ ⇒ log a = log n 1/ n

⇐ ⇒ log a = log n
n
log n n
⇐ ⇒ lim
n →∞ log a = lim n →∞

log n n = lim
Now, let f (x) = log( x) . Then lim
x n →∞ f (x) = 0 by L’Hopital’s Rule. Thus, lim n →∞ n →∞ f (n) = 0 also. So
lim
n →∞ a = lim n →∞ n 1/ n = 1.

Exercise 4. Show that ( cos z) ′ = − sin z and ( sin z) ′ = cos z.

Solution. We have by definition


(12
( e iz + e − iz)) ′ ( e iz − e − iz) ( e iz − e − iz)
(cos z) ′ = =i =−1 = − sin z
2 2i

and similarly
(12i
( e iz − e − iz)) ′ ( e iz + e − iz) ( e iz + e − iz)
(sin z) ′ = =i =1 = cos z.
2i 2

Exercise 5. Derive formulas (2.14).

Solution. Not available.

Exercise 6. Describe the following sets: {z : e z = i}, {z : e z = − 1}, { z : e z = − i}, {z : cos z = 0},
{ z : sin z = 0}.

Solution. Using the definition we obtain


{( 1 ) }
{ z : e z = i} = πi , { z : e z = − 1} = {(1 + 2 k) π i} ,
2+2k
{( − 1 ) } {( 1 ) }
{ z : e z = − i} = πi , { z : cos z = 0} = π ,
2+2k 2+k

and
{ z : sin z = 0} = { k π}

where k ∈ Z.

Exercise 7. Prove formulas for cos( z + w) and sin( z + w).

Solution. We have

cos( z) = e iz + e − iz
2

sin( z) = e iz − e − iz .
2i

27
1. Claim: cos( z) cos( w) − sin( z) sin( w) = cos( z + w). Proof:

e iw + e − iw e iw − e − iw
cos( z) cos( w) − sin( z) sin( w) = e iz + e − iz − e iz − e − iz
2 2 2i 2i
( e iz e iw + e − iz e − iw) 1 ( e iz e iw + e − iz e − iw)
=1 +
4 4

=1
2 e iz e iw + 1 2 e − iz e − iw
( e iz e iw + e − iz e − iw)
=1
2
= cos( z + w).

2. Claim: sin( z) cos( w) + cos( z) sin( w) = sin( z + w). Proof:

e iw + e − iw e iz + e − iz e iw − e − iw
sin( z) cos( w) + cos( z) sin( w) = e iz − e − iz +
2i 2 2 2i
( e iz e iw − e − iz e − iw) 1 ( e iz e iw − e − iz e − iw)
=1 +
4i 4i

=1
2 i e iz e iw − 1 2 e − iz e − iw
( e iz e iw − e − iz e − iw)
=1
2i
= sin( z + w).

Exercise 8. Define tan z = sin z


cos z ; where is this function defined and analytic?

Solution. Since both sin z and cos z are analytic in the entire complex plane, it follows from the discussion in the text following Definition
2.3 that tan z = sin z
cos z is analytic wherever cos z , 0. Now, cos z = 0 implies
that z is real and equal to an odd multiple of π 2 . Thus let

∣∣∣∣ k ∈ Z
}
G ≡ {( 2 k + 1) π .
2

Then tan z is defined and analytic on C − G. If z ∈ G, then cos z = 0 so tan z is undefined on the non-extended complex plane.

Exercise 9. Suppose that z n, z ∈ G = C − { z : z ≤ 0} and z n = r n e i θ n , z = re i θ where − π < θ, θ n < π. Prove that if z n → z then θ n → θ and r n → r.

Solution. Not available.

Exercise 10. Prove the following generalization of Proposition 2.20. Let G and Ω be open in C and suppose f and h are functions
defined on G, g : Ω → C and suppose that f (G) ⊂ Ω. Suppose that g and h are analytic, g ′( ω) , 0 for any ω, that f is continuous, h is
one-one, and that they satisfy h(z) = g( f (z)) for z in
G. Show that f is analytic. Give a formula for f ′( z).

Solution. Not available.

Exercise 11. Suppose that f : G → C is a branch of the logarithm and that n is an integer. Prove that z n = exp( n f (z)) for all z in G.

28
Solution. Let f (z) be a branch of the logarithm so that e f (z) = z. Let n ∈ Z and consider e n f (z). In the following cases we shall apply
several of the properties of the complex exponential function developed in the discussion on page 38 in the text. CASE 1: Assume n > 0.
Then we note that

e n f (z) = e f (z)+ f (z)+ ···+ f (z) (n times)

= e f (z) e f (z) · · · e f (z) ( n times)


( e f (z))n
=

= x n.

CASE 2: Assume n < 0. Then let m = − n so that m > 0 and

e n f (z) = 1
e m f (z) = 1 z m = z n,

the middle step following from Case 1. CASE 3: Assume n = 0. Then e n


f (z) = e 0 = 1 = z 0 = z n.

Exercise 12. Show that the real part of the function z 1 2 is always positive.

Solution. We know that we can write z = re i θ , 0, − π < θ < π and Log(z) = ln(r) + i θ. Thus

) ))
z 12 = eLog z 12 = e 1 2 Log z = e 12 ( ln r+i θ) = e 12 ln r e 12 θ i = e 12 ln r ( cos ( θ + i sin ( θ .
2 2

Hence,
)
Re(z) = e 12 ln r cos ( θ > 0,
2
)
since e 1 2 ln r > 0 and cos ( θ > 0 since − π < θ < π. Thus, the real part of the function z 12 is always positive.
2

Exercise 13. Let G = C − { z ∈ R : z ≤ 0} and let n be a positive integer. Find all analytic functions f : G → C such that z = ( f (z)) n for all z ∈
G.

Solution. Let Log(z) be the principal branch, then log( z) = Log(z) + 2 k π i for some k ∈ Z. Thus, we can write

f (z) = z 1/ n = e log( z)/n = e(Log( z)+ 2 k π i)/n = eLog( z)/n · e 2 k π i/n.

We know that the latter factor are the n-th roots of unity and depend only on k and n. They correspond to the n distinct powers of the
expression ζ = e 2 π i/n. Therefore, the branches of z 1/ n on the set U are given by

f (z) = ζ k · eLog( z)/n,

where k = 0, . . . , n − 1 and therefore they are all constant multiples of each other.

Exercise 14. Suppose f : G → C is analytic and that G is connected. Show that if f (z) is real for all z in G then f is constant.

Solution. First of all, we can write f : G → C as

f (z) = u(z) + iv(z)

29
where u, v are real-valued functions. Since f : G → C is analytic, that is f is continuously di ff erentiable (Definition 2.3), we have that u
and v have continuous partial derivatives. By Theorem 2.29, this implies that u, v satisfy the Cauchy-Riemann equations. That is,

∂u ∂u
and (3.1)
∂ x = ∂ v∂ y ∂ y = − ∂ v∂ x .

Since f (z) is real ∀ z ∈ G, this implies


v(z) ≡ 0

and therefore f (z) = u(z). So, since v(z) = 0, we have

∂v
∂ x = ∂ v∂ y = 0

and by ( 3.1 ) we obtain


∂u
∂ x = ∂ u∂ y = 0

and thus u ′( z) = 0 ( see reasoning of equation 2.22 and 2.23 on page 41). Hence f ′( z) = 0. Since G is connected and f : G → C is di ff erentiable
with f ′( z) = 0 ∀ z ∈ G, we have that f is constant.
{ ω : ω = exp ( 1 z )
Exercise 15. For r > 0 1et A = where 0 < | z| < r} ; determine the set A.

Solution. Define the set S = {z : 0 < | z| < r} where r > 0. The image of this set under 1/ z is clearly
{
T= z:1 .
r < |z|}

To find the image of A is the same as finding the image of T under e z.


Claim: The image of A is C − { 0} ( thus not depending on r). To proof the claim, we need to show that for w , 0, the equation e z = w has a
solution z ∈ T. Using polar coordinates we can write w = |w|e i θ. We have to find a complex number z = x + iy such that x 2 + y 2 > 1 r and e x e iy
= w = |w|e i θ. We want e x = | w| and y = θ + 2 k π, for some k ∈ Z. Using x = log | w| and k
0 such
that ( log | w|) 2 + ( θ + 2 k π) 2 > 1 r , then we found z = x + iy.

Exercise 16. Find an open connected set G ⊂ C and two continuous functions f and g defined on G such that f (z) 2 = g(z) 2 = 1 − z 2 for all
z in G. Can you make G maximal? Are f and g analytic?

Solution. Not available.

Exercise 17. Give the principal branch of √ 1 − z.

Solution. Not available.

Exercise 18. Let f : G → C and g : G → C be branches of z a and z b respectively. Show that f g is a branch of z a+b and f /g is a branch of z a −
b. Suppose that f (G) ⊂ G and g(G) ⊂ G and prove that both f ◦ g and g ◦ f are branches of z ab.

Solution. Not available.

Exercise 19. Let G be a region and define G ∗ = { z : ¯ z ∈ G}. If f : G → C is analytic prove that
f ∗ : G ∗ → C, defined by f ∗( z) = f ( ¯ x), is also analytic.

30
Solution. Let z = x + iy and let f (z) = u(x, y) + iv(x, y). By assumption f is analytic and therefore u and v have continuous partial
derivatives. In addition the Cauchy-Riemann Equations u x = v y and u y = − v x are satisfied. Since f ∗( z) = f ( ¯
x), we have f ∗( z) = u ∗( x, y) + iv ∗( x, y) where u ∗( x, y) = u(x, − y)
and v ∗( x, y) = − v(x, − y). Hence, we have u ∗ x( x, y) = u x( x, − y) = v y( x, − y), u ∗ y( x, y) = − u y( x, − y) = v x( x, − y), v ∗ x( x, y) = − v x( x, − y) and v ∗ y( x, y)
and therefore u ∗ x = v ∗ y and u ∗ y = − v ∗ x so f ∗ is analytic.

Exercise 20. Let z 1, z 2, . . . , z n be complex numbers such that Re z k > 0 and Re(z l . . . z k) > 0 for 1 ≤ k ≤ n. Show that log( z 1 . . . z n) = log z 1 + . . . + log z n, wh
log z is the principal branch of the logarithm. If the restrictions on the z k are removed, does the formula remain valid?

Solution. Let z 1, . . . , z n ∈ C such that Re(z j) > 0 and Re(z 1 · · · z j) > 0 for 1 ≤ j ≤ n. The proof will be by induction. Consider first the case
where n = 2. Let z 1, z 2 ∈ C as above. Then − π 2 < arg z 1 < π 2 , − π 2 <
arg z 2 < π 2 and − π 2 < arg( z 1 z 2) < π 2 . But note arg( z 1 z 2) = arg z 1 + arg z 2 implies that − π < arg z 1 + arg z 2 < π.
Now, recall

log( z 1 z 2) = ln | z 1 z 2| + i arg( z 1 z 2)

= ln | z 1| + ln | z 2| + i arg( z 1) + i arg( z 2)

= log( z 1) + log( z 2)

Assume this formula is true for n = k − 1. Let z 1, . . . , z k ∈ C as above. Then

log( z 1 . . . z k) = log(( z 1 · · · z k − 1) z k)

= log( z 1 · · · z k − 1) + log z k by the case when n = 2

= log z 1 + log z 2 + . . . + log z k − 1 + log z k by the case when n = k − 1

Hence, this is true for all n such that z i, 1 ≤ i ≤ n, satisfy the restrictions. If the restriction on the z k are removed, does the formula
remain valid? Consider the complex numbers z 1 = − 1 + 2 i, z 2 = − 2 + i. Then z 1 z 2 = 0 − 5 i. Clearly, z 1, z 2, and z 1 z 2 do not meet the
restrictions as stated above. Now,

log( z 1) = ln | z 1| + i arg z 1

log( z 2) = ln | z 2| + i arg z 2

Thus log z 1 + log z 2 = ln √ 5 + ln √ 5 + i( 3 π 2 ) = ln 5 + i( 3 π 2 ), but note that 3 π 2 < ( − π, π) and log( z 1 z 2) =


ln 5 + i( − π 2 ). Thus log z 1 + log z 2 , log( z 1 z 2) where log z is the principal branch of the logarithm. Hence, the formula is invalid.

Exercise 21. Prove that there is no branch of the logarithm defined on G = C − { 0}. ( Hint: Suppose such a branch exists and compare
this with the principal branch.)

Solution. Define the subset ˆ G of G by ˆ G = C − { z ∈ R : z ≤ 0}. We use the notation Log for the principal
part of the log on ˆ G, that is
Log(z) = log | z| + i arg( z)

where arg( z) ∈ (− π, π). We will prove the statement above by contradiction. Assume f (z) is a branch
of the logarithm defined on G. Then restricting f to ˆ G gives us a branch of the log
on ˆ G. Therefore its only di ff erence to the principal branch is 2 π ik for some k ∈ Z. This yields

f (z) = log | z| + i arg( z) + 2 π ik

where z ∈ ˆ G. Since f is analytic in G, it is continuous at − 1. But we can check that this is not the case, thus we have derived a
contradiction

31
3.3 Analytic functions as mappings. Möbius transformations
Exercise 1. 1. Find the image of {z : Re z < 0, | Im z| < π} under the exponential function.

Solution. We have

{ z : Re z < 0, | Im z| < π} = { z = x + iy : x < 0, − π < y < π}.

The image of {z = x + iy : x < 0, − π < y < π} under the exponential function is given by

{ e x+iy : x < 0, − π < y < π} = { e x e iy : x < 0, − π < y < π}

= { e x( cos( y) + i sin( y)) : x < 0, − π < y < π}

= { r( cos( y) + i sin( y)) : 0 < r < 1, − π < y < π}.

If 0 < r < 1 is fixed, then r( cos( y) + i sin( y)) describes a circle with radius r without the point ( − r, 0)
centered at ( 0, 0).
Since r varies between 0 and 1, we get that the image is a solid circle with radius 1, where the boundary does not belong to it, the
negative x-axis does not belong to it and the origin does not belong to is.

Exercise 2. Do exercise 1 for the set {z : |Im z| < π/ 2}.

Solution. We have
{ z : |Im z| < π/ 2} = { z = x + iy : x ∈ R, − π
2 < y < π 2 }.
The image of {z = x + iy : x ∈ R, − π 2 < y < π 2 } under the exponential function is given by

{ e x+iy : x ∈ R, − π
2 < y < π 2 } = { e x( cos( y) + i sin( y)) : x ∈ R, − π 2<y<π 2}

= { r( cos( y) + i sin( y)) : r > 0, − π


2 < y < π 2 }.
If r > 0 is fixed, then r( cos( y) + i sin( y)) describes a half circle with radius r centered at ( 0, 0) lying in the right half plane not touching the
imaginary axis.
Since r varies between 0 and infinity, we get that the image is the right half plane without the imaginary axis.

Exercise 3. Discuss the mapping properties of cos z and sin z.

Solution. Not available.

Exercise 4. Discuss the mapping properties of z n and z 1/ n for n ≥ 2. ( Hint: use polar coordinates.)

Solution. Not available.

Exercise 5. Find the fixed points of a dilation, a translation and the inversion on C ∞.

Solution. In general, we have

S (z) = az + b
cz + d .
To get a dilation S (z) = az, we have that b = 0, c = 0, d = 1 and a > 0. To find a fixed point, we have to find all z such that S (z) = z. In this
case az = z. Obviously z = 0 is a fixed point. Also z = ∞ is a fixed point, since a · ∞ = ∞( a > 0) or S ( ∞) = ac = a

0 = ∞.
To get a translation S (z) = z + b, we have that a = 1, c = 0, d = 1 and b ∈ R. To find a fixed point, we have to find all z such that S (z) = z. In
this case z +b = z, which is true if z = ∞. To see that S ( ∞) = ac = 10 = ∞.
To get a inversion S (z) = 1 z , we have that a = 0, b = 1, c = 1 and d = 0. To find a fixed point, we have to find all z such that S (z) = z. In this
case 1 z = z which is equivalent to z 2 = 1 and thus z = 1 and z = − 1 are fixed points.

32
Exercise 6. Evaluate the following cross ratios: (a) ( 7 + i, 1, 0, ∞) ( b) ( 2, 1 − i, 1, 1 + i) (c) ( 0, 1, i, − 1) ( d)
( i − 1, ∞, 1 + i, 0).

Solution. We have

S (z) = z − z 3 (3.2)
z − z 4 / z 2 z−2z−3 z 4 , if z 2, z 3, z 4 ∈ C

S (z) = z − z 3 (3.3)
z − z 4 , if z 2 = ∞

S (z) = z − z 3 (3.4)
z 2 − z 3 , if z 4 = ∞.

a) By ( 3.4 ) we get

(7 + i, 1, 0, ∞) = 7 + i − 0
1 − 0 = 7 + i.
b) By ( 3.2 ) we get

1+i
(2, 1 − i, 1, 1 + i) = 2 − 1
2 − 1 − i / 1 − i1
−−1i−1−i=1 1 − i / − −i 2 i = 2 1 − i = 2 (1 − i)(i + 1) = 21 + i 2 = 1 + i.

c) By ( 3.2 ) we get

1+i
(0, 1, i, − 1) = 0 − i
0 + 1/ 1 −1i + 1 = − i/ 1 − i 2 = − i 2 1−i 1 + i = − i 2 2(1 + i) = − i − i 2 = 1 − i.

d) By ( 3.3 ) we get

i+1
( i − 1, ∞, 1 + i, 0) = i − 1 − 1 − i
i−1−0=−2 i−1 i + 1 = − 2−
2 − 2( i + 1) = 1 + i.

Exercise 7. If Tz = az+b cz+d , find z 2, z 3, z 4 ( in terms of a, b, c, d) such that Tz = (z, z 2, z 3, z 4).

Solution. The inverse of T is given by

T − 1( z) = dz − b
− cz + a

as shown on p. 47. We have T − 1( 1) = d − b a−c, T − 1( 0) = − ba and T − 1( ∞) = − dc . Set z 2 = d − b a−c, z 3 = − ba and


z 4 = − dc to obtain Tz = (z, z 2, z 3, z 4).

Exercise 8. If Tz = az+b cz+d show that T( R ∞) = R ∞ i ff we can choose a, b, c, d to be real numbers.

Solution. Let Tz = az + b
cz + d . Assume T( R ∞) = R ∞. Then let z 0 ∈ R ∞ such that Tz 0 = 0. Observe that this

implies az 0 = − b, so − b
a ∈ R ∞ and r 1 ≡ b a ∈ R ∞ as well. Likewise, if z ∞ ∈ R ∞ such that Tz ∞ = ∞, then

33
r2≡ d
c ∈ R ∞. Now let z 1 ∈ R ∞ such that Tz 1 = 1. Then
az 1 + b cz 1 + d = 1

az 1 + b = cz 1 + d z 1
(1−c )
=d
a a−b a
z1
c − z1 a − r2 a + r1 c = 0
z1+ r1
c = z1+ r2 a
z1+ r1
z1+ r2= c a ∈ R ∞.

Let r 3 = c
a . Then d a = d c × c a = r 2 r 3 ∈ R ∞. Thus

Tz = az + b = z + r1
cz + d = z + bac a z + da r3 z + r2 r3

and we have thus found real coe ffi cients for T.


Now conversely, let us assume that a, b, c, d ∈ R ∞. Thus, for all z ∈ R ∞, we must have Tz ∈ R ∞ due to the fact that R is closed under
multiplication and addition. Thus, T( R ∞) = R ∞.

Exercise 9. If Tz = az+b cz+d , find necessary and su ffi cient conditions that T( Γ) = Γ where Γ is the unit circle
{ z : |z| = 1}.

Solution. We want if z ¯ z = 1, then T(z)T(z) = 1.

az + b cz + d = 1 ⇐⇒ ( az + b)( ¯ a zz̄¯ + ¯b)


T(z)T(z) = 1 ⇐⇒ az + b
cz + d ( cz + d)( ¯c zz̄¯ + ¯d) = 1
⇐ ⇒ z̄¯ + ¯dcz + d ¯ d
ā zz̄¯ + b ¯ a zz̄¯ + az ¯ b + b ¯ b = cz ¯ c zz̄¯ + d ¯ c z
az ¯a

⇐ ⇒ ¯ ¯a − c ¯ c) + z(a ¯ b − c ¯
z z(a
z̄(a d) + ¯ z(b ¯a + d ¯ c) + b ¯ b − d ¯ d = 0.

This is the same as the equation z ¯ z − 1 = 0 if

a bb̄¯ − c ¯ d = 0

ā¯ − d ¯ c = 0
ba

and

ā¯ − c ¯ c = 1
aa

b bb̄¯ − d ¯ d = − 1

which is equivalent to |a| 2 − | c| 2 = 1 and 1 = | d| 2 − | b| 2. Hence, we have the two conditions

a bb̄¯ − c ¯ d = 0 and | a| 2 + | b| 2 = | c| 2 + | d| 2.


These are the su ffi cient conditions. Let c = λ ¯ b, then a ¯ b−c¯ d = 0 yields a ¯ b − λ ¯ b ¯d
d̄ = 0 i ff a = λ ¯ d i ff d =¯
λ.
Insert this into |c| 2 + | d| 2 = | a| 2 + | b| 2 yields

| λ| 2| b| 2 + | a| 2
| λ| 2 = | a| 2 + | b| 2,

34
and therefore we can take | λ| = 1. Then 1¯
λ= λ, and so the form of the Möbius transformation is

T(z) = az + b where | λ| = 1
b̄z + ¯ a) ,
λ( ¯bz

or
T(z) = ¯ λ az + b where | λ| = 1.
b̄z + ¯ a ,
bz

Take λ = e − i θ, then

T(z) = e i θ az + b
b̄z + ¯ a , for some θ.
bz

This mapping transforms |z| = 1 into |T(z)| = 1.

Exercise 10. Let D = {z : |z| < 1} and find all Möbius transformations T such that T(D) = D.

Solution. Take an α ∈ D = {z : |z| < 1} such that T( α) = 0. Its symmetric point with respect to the unit circle is

α∗= 1
ᾱ
since

z∗− a = R2
zz̄ − ¯ a
(see page 51) with a = 0 and R = 1 ( the unit circle). Therefore T( α ∗) = ∞. Thus T looks like

T(z) = K z − α
ᾱ z − 1
α

where K is a constant. (It is easy to check that T( α) = 0 and T( α ∗) = T( 1¯ α)= ∞). Finally, we are going to
choose the constant K in such a way that |T(z 0)| = 1 where z 0 = e i θ. We have

T(z 0) = K e i θ − α
ᾱ e i θ − 1
α

and therefore
( e i θ − α) ( e − i θ − ¯ ( e i θ − α) ( e − i θ − ¯
α) α)
1 = | T(z 0)| = | K| |e i θ − α| = | K| (¯
ᾱ¯ e i θ − 1| = | K|
|α | e i θ| · | α
ᾱ¯ − e − i θ| α − e − i θ) ( α − e i θ) = | K|.

So |K| = 1 implies K = e i θ for some real θ. We arrive at

T(z) = e i θ z − α for some real θ.


ᾱ z − 1 ,
α

Exercise 11. Show that the definition of symmetry (3.17) does not depend on the choice of points z 2, z 3, z 4.
That is, show that if ω 2, ω 3, ω 4 are also in Γ then equation (3.18) is satisfied i ff ( z ∗, ω 2, ω 3, ω 4) = ( z, ω 2, ω 3, ω 4).
(Hint: Use Exercise 8.)

Solution. Let Γ be a circle in C ∞ containing points z 2, z 3, z 4, w 2, w 3, and w 4, with all z i distinct and all w i
distinct. Let z ∈ C ∞ and consider z ∗, as defined in the text and established by the points z i. We know that

( z ∗, z 2, z 3, z 4) = ( z, z 2, z 3, z 4)

35
Recall that the above left-hand cross-ratio implies a unique Möbius transformation T for which Tz 2 = 1,
Tz 3 = 0, and Tz 4 = ∞. By Proposition 3.10, T maps Γ to R ∞. In fact, the definition of symmetry may be rewritten as

Tz ∗ = Tz.

or
z ∗ = T − 1 Tz (3.5)

Likewise, there is some transformation S for which Sw 2 = 1, Sw 3 = 0, and Sw 4 = ∞. Again, S maps Γ to


R ∞. We wish to show that
z ∗ = S − 1 S z. (3.6)

Now, we proceed by showing that the right hand sides of ( 3.5 ) and ( 3.6 ) must be equal:

T − 1 Tz = T − 1 TS − 1 S z

Observe here that TS − 1 must have real coe ffi cients by Exercise 8 because R ∞ 7→ R ∞ under TS − 1. Thus we observe that TS − 1 = TS − 1 so

T − 1 Tz = T − 1 TS − 1 S z = S − 1 S z.

Hence we have the desired result.

Exercise 12. Prove Theorem 3.4.

Solution. Not available.

Exercise 13. Give a discussion of the mapping f (z) = 12 ( z + 1/ z).


( z +1z )
Solution. The function f (z) = 12 = z 2+ 21 z can be defined for all z ∈ C − { 0} and therefore also in the
punctured disk 0 < | z| < 1. To see that in this domain the function is injective, let z 1, z 2 be two numbers in the domain of f with the same
image, then

0 = f (z 1) − f (z 2) = z 21 + 1 = ( z 1 z 2 − 1)( z 1 − z 2) .
z 1 − z 22 + 1 z 2 = z 21 z 2 + z 2 − z 1zz1 22
z 2− z 1 z1 z2

With the assumption 0 < | z i| < 1, i = 1, 2 the factor z 1 z 2 − 1, is always nonzero and we conclude that z 1 = z 2;
hence f (z) is injective. The range of the function is C − { z ∈ C | < z ∈ [− 1, 1] and Im z = 0}. To see this, write z = re i θ in polar coordinates and
let f (z) = w = a + ib, a, b ∈ R.

( ) [( ) ( ) ]
f (z) = f (re i θ) = 1 re i θ + 1 =1 r+1 cos θ + i r−1 sin θ .
2 r e−iθ 2 r r

For the real and imaginary part of w the following equations must hold
( r +1r )
a =1 2
cos θ

( r − 1r ) (3.7)
b =1 2
sin θ.

If f (z) = w has imaginary part b = 0 then sin θ = 0 and | cos θ| = 1. Therefore points of the form a + ib, a ∈ [− 1, 1], b = 0 cannot be in the
range of f . For all other points the equations ( 3.7 ) can be solved for r and θ uniquely (after restricting the argument to [ − π, π)).

Given any value of r ∈ ( 0, 1), the graph of f (re i θ) as a function of θ looks like an ellipse. In fact from

36
)2 (b )2
formulas ( 3.7 ) we see that ( a 12 ( r+ 1 r ) + 12 ( 1 − 1r) = 1.

If we fix the argument θ and let r vary in ( 0, 1) it follows from from equation ( 3.7 ) that the graph of f (re i θ)
is a hyperbola and it degenerates to rays if z is purely real or imaginary. In the case θ ∈ {( 2 k + 1) π | k ∈ Z}
the graph of f in dependence on r is on the imaginary axis and for θ ∈ { 2 k π | k ∈ Z} the graph of f (re i θ) is either ( −∞, − 1) or ( 1, ∞). If cos θ , 0
)2− (b ) 2 = 1.
and sin θ , 0 then ( a
cos θ sin θ

Exercise 14. Suppose that one circle is contained inside another and that they are tangent at the point a. Let G be the region between
the two circles and map G conformally onto the open unit disk. (Hint: first try
( z − a) − 1.)

Solution. Using the hint, define the Möbius transformation T(z) = (z − a) − 1 which sends the region G between two lines. Afterward
applying a rotation followed by a translation, it is possible to send this region to any region bounded by two parallel lines we want.
Hence, choose S (z) = cz + d where |c| = 1 such that
{ x + iy : 0 < y < π }
S (T(G)) = .
2

Applying the exponential function to this region yields the right half plane

exp( S (T(G))) = {x + iy : x > 0}.

Finally, the Möbius transformation

R(z) = z − 1
z+1
maps the right half plane onto the unit disk (see page 53). Hence the function f defined by R( exp( S (T(z)))) maps G onto D and is the
desired conformal mapping ( f is a composition of conformal mappings). Doing some simplifications, we obtain

z−a+ d− 1
f (z) = e c
e cz − a + d + 1

where the constants c and d will depend on the circle location.

Exercise 15. Can you map the open unit disk conformally onto {z : 0 < | z| < 1}?

Solution. Not available.

Exercise 16. Map G = C − { z : − 1 ≤ z ≤ 1} onto the open unit disk by an analytic function f . Can f be one-one?

Solution. Not available.

Exercise 17. Let G be a region and suppose that f : G → C is analytic such that f (G) is a subset of a circle. Show that f is constant.

Solution. Not available.

Exercise 18. Let −∞ < a < b < ∞ and put Mz = z − ia z − ib . Define the lines L 1 = { z : Im z = b}, L 2 = { z : Im z =
a} and L 3 = { z : Re z = 0}. Determine which of the regions A, B, C, D, E, F in Figure 1, are mapped by M onto the regions U, V, W, X, Y, Z in
Figure 2.

37
Solution. We easily see that we have M(ia) = 0. Therefore the region B, C, E and F which touch the line ia are mapped somehow to the
region U, V, X and Y which touch 0. Similarly we have M(ib) = ∞ and therefore the region B and E which touch the line ib are mapped
somehow to the region U and X which touch ∞. Thus we conclude that C or F goes to either V or Y. Let us find out. Let x, y be small
positive real numbers such that the poin z = x + iy + ia ∈ E. Thus, the imaginary part of Mz is a positive number multiplied by x(b − a) and
therefore also positive. Therefore we conclude that M maps E to U and B to X. Because B and C meet at the line ia, we conclude that
X and M(C) do, too. Hence, M maps C to Y and F to V. By a similar argument, we obtain that M maps A to Z and D to W.

Exercise 19. Let a, b, and M be as in Exercise 18 and let log be the principal branch of the logarithm. (a) Show that log( Mz) is defined
for all z except z = ic, a ≤ c ≤ b; and if h(z) = Im [ log Mz] then
0 < h(z) < π for Re z > 0.
(b) Show that log( z − ic) is defined for Re z > 0 and any real number c; also prove that |Im log( z − ic)| < π 2
if Re z > 0.
(c) Let h be as in (a) and prove that h(z) = Im [ log( z − ia) − log( z − ib)]. (d) Show that

∫b
dt z − it = i[ log( z − ib) − log( z − ia)]

(Hint: Use the Fundamental Theorem of Calculus.) (e) Combine


(c) and (d) to get that
∫b ) (y−b )
x x 2 + ( y − t) 2 dt = arctan (y − a
h(x + iy) = − arctan
a x x

(f) Interpret part (e) geometrically and show that for Re z > 0, h(z) is the angle depicted in the figure.

Solution. Not available.

Exercise 20. Let S z = az+b cz+d and Tz = α z+ β γ z+ δ , show that S = T i ff there is a non zero complex number λ such
that α = λ a, β = λ b, γ = λ c, δ = λ d.

Solution. ⇐: Let λ , 0 be a complex number such that

α=λa

β=λb

γ=λc

δ = λ d.

Then
T(z) = α z + β
γ z + δ = λ azλ +
czλ+bλ d = λ( az
λ( +czb)+ d) = (az +( cz
b) + d) = S (z).
Thus, S = T.
⇒ : Let S = T, that is S (z) = T(z). Then S ( 0) = T( 0), S ( 1) = T( 1) and S ( ∞) = T( ∞) which is equivalent to

bd=β
(3.8)
δ = λ 1 ⇐⇒ b = λ 1 d, β = λ 1 δ
a+bc+d=α+β
(3.9)
γ+δ
ac=α
(3.10)
γ = λ 3 ⇐⇒ a = λ 3 c, α = λ 3 γ.

38
Insert b = λ 1 d, β = λ 1 δ, a = λ 3 c and α = λ 3 γ into ( 3.9 ), then

λ3 c + λ1 d
c + d = λ 3 γ + λ 1γδ+ δ
⇐ ⇒ λ3 c γ + λ1 d γ + λ3 c δ + λ1 d δ = λ3 γ c + λ1 δ c + λ3 γ d + λ1 δ d

⇐ ⇒ λ1 d γ + λ3 c δ − λ1 δ c − λ3 γ d = 0

⇐ ⇒ λ 1( d γ − δ c) − λ 3( d γ − δ c) = 0

⇐ ⇒ ( λ 1 − λ 3)( d γ − δ c) = 0.

Thus, either λ 1 − λ 3 = 0 or d γ − δ c = 0.
If λ 1 − λ 3 = 0, then from ( 3.8 ) and ( 3.9 ), we get λ 3 = ac = b d = λ 1 which implies ad − bc = 0 a contradiction
(Not possible, otherwise we do not have a Möbius transformation). Thus, d γ − δ c = 0 ⇐⇒
c
d=γ δ ⇐⇒ γ c = δ d := λ, λ , 0 or

γ=cλ (3.11)

δ = d λ. (3.12)

Insert ( 3.11 ) and ( 3.12 ) into ( 3.8 ) to obtain

bd=β
δ ⇒ b d = β d λ ⇒ β = λ b.

Insert ( 3.11 ) and ( 3.12 ) into ( 3.10 ) to obtain

ac=α
γ ⇒ a c = α c λ ⇒ α = λ a.

Therefore, we have
α = λ a, β = λ b, γ = λ c, δ = λ d.

Exercise 21. Let T be a Möbius transformation with fixed points z 1 and z 2. If S is a Möbius transformation show that S − 1 TS has fixed
points S − 1 z 1, and S − 1 z 2.

Solution. To show
S − 1 TS (S − 1 z 1) = S − 1 z 1.

We have
S − 1 TS (S − 1 z 1) = S − 1 TS S − 1 z 1 = S − 1 Tz 1 = S − 1 z 1,

where the first step follows by the associativity of compositions, the second step since S S − 1 = id and the last step by Tz 1 = z 1 since z 1 is
a fixed point of T. To show

S − 1 TS (S − 1 z 2) = S − 1 z 2.

We have
S − 1 TS (S − 1 z 2) = S − 1 TS S − 1 z 2 = S − 1 Tz 2 = S − 1 z 2,

where the first step follows by the associativity of compositions, the second step since S S − 1 = id and the last step by Tz 2 = z 2 since z 2 is
a fixed point of T. Note that S − 1 TS is a well defined Möbius transformation, since S and T are Möbius transformation. The
composition of Möbius transformation is a Möbius transformation.

39
Exercise 22. ( a) Show that a Möbius transformation has 0 and ∞ as its only fixed points i ff it is a dilation, but not the identity.

(b) Show that a Möbius transformation has ∞ as its only fixed point i ff it is a translation, but not the identity.

Solution. a) Let M be a Möbius transformation with exactly two fixed points, at 0 and ∞. We know that

Mz = az + b
cz + d

with a, b, c, d ∈ C. We know that M( 0) = 0 so b


d = 0, whence we conclude that b = 0. But we also
know that M( ∞) = ∞ so a
c = ∞, meaning c = 0. Thus Mz = az d = α z for some α ∈ C. Since both
b and c were 0, we know that both a and d are nonzero so α is likewise nonzero and finite. Suppose by way of contradiction
that α = 1. Then for any ˜ z ∈ C ∞, M ˜ z = ˜ z and M has infinitely many fixed
points, a contradiction. Thus α , 1 and M is a (complex) dilation not equal to the identity. On the other hand, assume M is a
dilation not equal to the identity. In this case, we are free to express M as Mz = α z with α , 1. It is clear from this representation
that M has fixed points at 0 and ∞ and, of course, at no other points.

b) Let M be a Möbius transformation with exactly one fixed point at ∞. Again, we know that

Mz = az + b
cz + d

and we can see that c = 0 as before. This time, however, we also examine the lack of a second fixed point. Recall that if ˜
z is a fixed point of M, then ˜ z satisfies c ˜ z 2 + ( d − a) ˜ z − b = 0, which in the case

of c = 0 reduces to (d − a) ˜ z − b = 0 or ˜ z=b
d − a . We note that b , 0 because 0 is not a fixed point
of M. Suppose that d , a. In such a case, there is a finite value of ˜ z that is a fixed point of M, a
contradiction. Thus, it must be the case that d = a and

Mz = z + b
d.

Thus M is a translation.

Now assume M is a translation. Then we can express M as Mz = z + β. Here, in the convention of the text, we have a = 1, b = β, c
= 0, and d = 1. The fixed points ˜ z of M satisfy c ˜ z 2 + ( d − a) ˜ z − b = 0,
but no finite ˜ z satisfies this equation for the given coe ffi cients. Clearly, however, M has ∞ as a fixed
point, so ∞ is the only fixed point of M.

Exercise 23. Show that a Möbius transformation T satisfies T( 0) = ∞ and T( ∞) = 0 i ff Tz = az − 1 for some a in C.

Solution. Not available.

Exercise 24. Let T be a Möbius transformation, T , the identity. Show that a Möbius transformation S commutes with T if S and T have
the same fixed points. (Hint: Use Exercises 21 and 22.)

Solution. Let T and S have the same fixed points. To show TS = ST, T , id.

? : Suppose T and S have two fixed points, say z 1 and z 2. Let M be a Möbius transformation with M(z 1) = 0
and M(z 2) = ∞. Then
MS M − 1( 0) = MS M − 1 Mz 1 = MS z 1 = Mz 1 = 0

40
and
MS M − 1( ∞) = MS M − 1 Mz 2 = MS z 2 = Mz 2 = ∞.

Thus MS M − 1 is a dilation by exercise 22 a) since MS M − 1 has 0 and ∞ as its only fixed points. Similar, we obtain MTM − 1( 0) = 0 and
MTM − 1( ∞) = ∞ and therefore is also a dilation. It is easy to check that dilations commute (define C(z) = az, a > 0 and D(z) = bz, b > 0, then
CD(z) = abz = baz = DC(z)), thus

( MTM − 1)( MS M − 1) = ( MS M − 1)( MTM − 1)

⇒ MTS M − 1 = MSTM − 1
⇒ TS = ST.

? : Suppose T and S have one fixed points, say z. Let M be a Möbius transformation with M(z) = ∞. Then

MS M − 1( ∞) = MS M − 1 Mz = MS z = Mz = ∞.

Thus MS M − 1 is a translation by exercise 22 b) since MS M − 1 has ∞ as its only fixed point. Similar, we obtain MTM − 1( ∞) = ∞ and
therefore is also a translation. It is easy to check that translations commute (define C(z) = z + 1 and D(z) = z + b, b > 0, then CD(z) = z +
a + b = z + b + a = DC(z)), thus

( MTM − 1)( MS M − 1) = ( MS M − 1)( MTM − 1)

⇒ MTS M − 1 = MSTM − 1
⇒ TS = ST.

Exercise 25. Find all the abelian subgroups of the group of Möbius transformations.

Solution. Not available.

Exercise 26. 26. (a) Let GL 2( C) = all invertible 2 × 2 matrices with entries in C and let M be the group of Möbius transformations. Define
(ab )
ϕ : GL 2( C) → M by ϕ = az+b
cz+d . Show that ϕ is a group
cd
homomorphism of GL 2( C) onto M. Find the kernel of ϕ.
(b) Let S L 2( C) be the subgroup of GL 2( C) consisting of all matrices of determinant 1. Show that the image of S L 2( C) under ϕ is all of M. What
part of the kernel of ϕ is in S L 2( C)?

Solution. a) We have to check that if


(ab ) (ˆ )
aˆ b
A= and B =
cd ĉ dˆ

then ϕ( AB) = ϕ( A) ◦ ϕ( B). A simple calculation shows that this is true. To find the kernel of the group homomorphism we have to find all
z such that az+b
cz+d = z. This is equivalent to az + b = cz 2 + dz and by
comparing coe ffi cients we obtain b = c = 0 and a = d. Therefore, the kernel is given by N = ker( ϕ) = {λ I :
λ ∈ C ×}. Note that the kernel is a normal subgroup of GL 2( C).
b) Restricting ϕ to S L 2( C) still yields a surjective map since for any matrix A ∈ GL 2( C) both A and the modification M = 1
√ det A A have the same image and the modification matrix M has by construction

determinant 1. The kernel of the restriction is simply N ∩ S L 2( C) = { ± I}.

Exercise 27. If G is a group and N is a subgroup then N is said to be a normal subgroup of G if S − 1 TS ∈ N


whenever T ∈ N and S ∈ G. G is a simple group if the only normal subgroups of G are {I} (I = the identity of G) and G itself. Prove that the
group M of Möbius transformations is a simple group.

41
Solution. Not available.

Exercise 28. Discuss the mapping properties of ( 1 − z) i.

Solution. Not available.

Exercise 29. For complex numbers α and β with | α| 2 + | β| 2 = 1

β
u α,β = α z − ¯ and let U = {u α,β : | α| 2 + | β| 2 = 1}
β z + ¯α

(a) Show that U is a group under composition. (b) If SU 2 is the set of all unitary matrices with determinant 1, show that SU 2 is a group
under matrix multiplication and that for each A in SU 2 there are unique complex numbers α and β with | α| 2 + | β| 2 = 1

and (αβ )
A=
− ¯ββ̄ ¯ α
(αβ )
(c) Show that 7→ u α,β is an isomorphism of the group SU 2 onto U. What is its kernel?
− ¯ββ̄ ¯ α
(d) If l ∈ { 0, 12 , 1, 3 2 , . . .} let H l = all the polynomials of degree ≤ 2 l. For u α,β = u in U define T( l) u: H l → H l,
by (T( l) u f )(z) = ( β z + ¯ α) 2 l f (u(z)). Show that T( l) u is an invertible linear transformation on H l, and u 7→ T( l) u

is an injective homomorphism of U into the group of invertible linear transformations of H l, onto H l.

Solution. Not available.

Exercise 30. For |z| < l define f (z) by


• [ (1+z ) ] 1/2 •••••••
••

f (z) = exp •• − i log i
• 1−z

(a) Show that f maps D = {z : |z| < 1} conformally onto an annulus G. (b) Find all Möbius transformations S (z) that map D onto D and
such that f (S (z)) = f (z) when |z| < 1.

Solution. Not available.

42
Chapter 4

Complex Integration

4.1 Riemann-Stieltjes integrals


Exercise 1. Let γ : [ a, b] → R be non decreasing. Show that γ is of bounded variation and V( γ) = γ( b) − γ( a).

Solution. Let γ : [ a, b] → R be a monotone, non-decreasing function. For a given partition of [a, b],
{ a = t 0 < t 1 < . . . < t m = b}, for all n ∈ N, γ( t n) ≥ γ( t n − 1). Therefore, | γ( t n) − γ( t n − 1)| = γ( t n) − γ( t n − 1) ≥ 0.
Let P = {a = t 0 < t 1 < . . . < t m = b}, any partition of [a, b]. Then,

∑m
| γ( t n) − γ( t n − 1)| = [ γ( t m) − γ( t m − 1)] + [ γ( t m − 1) − γ( t m − 2)] + . . . + [ γ( t 2) − γ( t 1)] + [ γ( t 1) − γ( t 0)]
n= 1

= [ γ( b) − γ( t m − 1)] + [ γ( t m − 1) − γ( t m − 2)] + . . . + [ γ( t 2) − γ( t 1)] + [ γ( t 1) − γ( a)]

= γ( b) − γ( a)

Since γ( a), γ( b) ∈ R, γ( b) − γ( a) < ∞. It follows that γ is of bounded variation for any partition P. And also v( γ; P) = sup{v( γ; P) : P a partition
of [a, b]} = V( γ) = γ( b) − γ( a).

Exercise 2. Prove Proposition 1.2.

Solution. Let γ : [ a, b] → C be of bounded variation. (a) If P and Q are partitions of [a, b] and P ⊂ Q then v( γ; P) ≤ v( γ; Q). Proof will be by
induction. Let P = {a = t 0 < t 1 < . . . < t m = b}. Suppose Q = P ∪ { x}, where t k < x < t k+ 1.

Then,

v( γ; P) = | γ( t i) − γ( t i − 1)| + | γ( t k+ 1) − γ( t k)|
i,k+ 1

≤∑ | γ( t i) − γ( t i − 1)| + | γ( x) − γ( t k)| + | γ( t k+ 1) − γ( x)|


i,k+ 1

= v( γ; Q)

Now consider the general case on the number of elements of Q \ P. The smallest partition set for Q \ P contains the trivial partition
composed of {a, b}. Using the case defined above, we have v( γ; Q) ≥ v( γ; P) ≥
v( γ; Q \ P) ≥ | γ( b) − γ( a)|, whenever P ⊂ Q. (b) If σ : [ a, b] → C is also of bounded variation and α, β ∈ C, then αγ + βσ is of bounded
variation and

43
V( αγ + βσ) ≤ | α| V( γ) + |β| V( σ).
Let P = {a = t 0 < t 1 < . . . < t m = b} be a partition. Since γ and σ are of bounded variation for [a, b] ⊂ R,
∑m
there exists constants M γ > 0 and M σ > 0, respectively, such that v( γ; P) = | γ( t k) − γ( t k − 1)| ≤ M γ and
k= 1
∑m
v( γ; P) = | σ( t k) − σ( t k − 1)| ≤ M σ. Then,
k= 1

∑m
v( αγ + βσ; P) = |( αγ + βσ)( t k) − ( αγ + βσ)( t k − 1)|
k= 1

∑m
= | αγ( t k) + βσ( t k) − αγ( t k − 1) − βσ( t k − 1)|
k= 1

∑m
= |( αγ( t k) − αγ( t k − 1)) + ( βσ( t k) − βσ( t k − 1))|
k= 1


≤m | αγ( t k) − αγ( t k − 1)| + | βσ( t k) − βσ( t k − 1)|
k= 1

∑m ∑m
= | αγ( t k) − αγ( t k − 1)| + | βσ( t k) − βσ( t k − 1)|
k= 1 k= 1

∑m ∑m
= | α| |(γ( t k) − γ( t k − 1))| + | β| |(σ( t k) − σ( t k − 1))|
k= 1 k= 1

∑ ∑
= | α| m | γ( t k) − γ( t k − 1)| + | β| m | σ( t k) − σ( t k − 1)|
k= 1 k= 1

= | α| v( γ; P) + | β| v( σ; P)

≤ | α| sup{v( γ; P) : P a partition of [a, b]} + | β| sup{v( σ; P) : P a partition of [a, b]}

= | α| V( γ) + |β| V( σ) < ∞

The result is two-fold, V( αγ + βσ) ≤ | α| V( γ) + |β| V( σ) and it follows αγ + βσ is of bounded variation by


| α| V( γ) + |β| V( σ).

Exercise 3. Prove Proposition 1.7.

Solution. Not available.

Exercise 4. Prove Proposition 1.8 (Use induction).

Solution. Not available.

Exercise 5. Let γ( t) = exp(( − 1 + i)/t − 1) for 0 < t ≤ 1 and γ( 0) = 0. Show that γ is a rectifiable path and find V( γ). Give a rough sketch of the
trace of γ.

Solution. Not available.

Exercise 6. Show that if γ : [ a, b] → C is a Lipschitz function then γ is of bounded variation.

44
Solution. Let γ : [ a, b] → C be Lipschitz, that is ∃ a constant C > 0 such that

| γ( x) − γ( y)| ≤ C|x − y|, ∀ x, y ∈ [ a, b].

Then, for any partition P = {a = t 0 < t 1 < . . . < t m = b} of [a, b], we have

∑m ∑ ∑m
v( γ; P) = | γ( t k) − γ( t k − 1)| ≤ m C|t k − t k − 1| = C | t k − t k − 1| = C(b − a) =: M > 0.
k= 1 k= 1 k= 1

Hence, the function γ : [ a, b] → C, for [a, b] ⊂ R, is of bounded variation, since there is a constant M > 0
such that for any partition P = {a = t 0 < t 1 < . . . < t m = b} of [a, b]

∑m
v( γ; P) = | γ( t k) − γ( t k − 1)| ≤ M.
k= 1

Exercise 7. Show that γ : [ 0, 1] → C, defined by γ( t) = t + it sin 1 t for t , 0 and γ( 0) = 0, is a path but is not rectifiable. Sketch this path.

Solution. Not available.

Exercise

8. Let γ and σ be the two polygons [ 1, i] and [ 1, 1 + i, i]. Express γ and σ as paths and calculate

γf and ∫ σf where f (z) = |z| 2.

Solution. For γ we have γ( t) = ( 1 − t) + it for 0 ≤ t ≤ 1. So γ ′( t) = − 1 + i. Therefore,

∫ ∫1
| z| 2 dz = ((1 − t) 2 + t 2)( − 1 + i)dt
γ 0
∫1
= ( − 1 + i) (2 t 2 − 2 t + 1) dt
0
[ 2 3 t3− t2+ t ]1
= ( − 1 + i)
0
(23−1+1 )
= ( − 1 + i)

=−2
3 + i 23 .

Since σ is composed of multiple lines, the path is given by { γ 1( t) = 1 + it : 0 ≤ t ≤ 1; γ 2( t) = ( 1 − t) + i :

45
0 ≤ t ≤ 1}. So γ ′
1( t) = i and γ ′ 2( t) = − 1. Therefore,
∫ ∫ ∫
f= f+ f
γ γ1 γ2
∫ ∫
= | z| 2 dz + | z| 2 dz
γ1 γ2
∫1 ∫1
= ( t 2 + 1)( i)dt + ((1 − t) 2 + 1)( − 1) dt
0 0
∫1
=i ( t 2 + 1) dt − ∫ 1 ( t 2 − 2 t + 2) dt
0 0
[ t3 ]1 ]1
=i − [ t3
3+t 0
3 − t2+ 2 t 0
)
=4
3i−(13−1+2

=4
3i−4 3

=4
3( − 1 + i)

Exercise 9. Define γ : [ 0, 2 π] → C by γ( t) = exp( int) where n is some integer (positive, negative, or zero). Show that ∫
1 z dz = 2 π in.
γ

Solution. Clearly, γ( t) = e int is continuous and smooth on [ 0, 2 π]. Thus

∫ ∫ 2π ∫ 2π
z − 1 dz = e − int ine int dt = in dt = in( 2 π − 0) = 2 π in.
γ 0 0

Exercise 10. Define γ( t) = e it for 0 ≤ t ≤ 2 π and find ∫


γ z n dz for every integer n.

Solution. Clearly, γ( t) = e int is continuous and smooth on [ 0, 2 π] ( It is the unit circle). Case 1: n = − 1

∫ ∫ 2π ∫ 2π
z − 1 dz = e − it ie it dt = i dt = 2 π i.
γ 0 0

Case 2: n , − 1
∫ ∫ 2π ∫ 2π [ e i(n+ 1) t ]2π
z n dz = e int ie it dt = i e i(n+ 1) t dt = t
γ 0 0 i(n + 1) 0

1 [ e i(n+ 1) t] 2 π [ e i(n+ 1)2 π − 1]


=
n+1 0= 1 n+1
1
=
n + 1 [cos(( n + 1)2 π) − i sin(( n + 1)2 π) 1] = 1 n + 1 [1 − i · 0 − 1]
= 0.

1 z dz.
Exercise 11. Let γ be the closed polygon [ 1 − i, 1 + i, − 1 + i, − 1 − i, 1 − i]. Find ∫
γ

46

Você também pode gostar